McNeil's - surgery Flashcards

1
Q

All of the following increase the incidence of hernia/wound dehiscence EXCEPT

a. wound infection
b. steroids
c. pulmonary complications
d. previous surgery
e. NIDDM

A

D?

[Incisional hernias] occur as a result of excessive tension and inadequate healing of a previous incision, which is often associated with surgical site infections. These hernias enlarge over time, leading to pain, bowel obstruction, incarceration, and strangulation. Obesity, advanced age, malnutrition, ascites, pregnancy, and conditions that increase intra-abdominal pressure are predisposed factors to the development of an incisional hernia.
- Sabiston

Poor surgical technique is the worst, wound infection second worst, then age, then comorbidities, then obesity … See Way p793

The incidence of wound disruption ranges from 0.5 to 3.0% and averages about 2% in abdominal operations. The incidence rises to 5% in elderly patients, which suggests that coexisting diseases or the integrity of the fascial tissue itself is also an important factor. Other predisposing factors include malnutrition, obesity, acquired or pharmacologic immune deficiency, diabetes, renal insufficiency, and advanced malignancy. Local wound events, such as infection, hematoma, and ischemia, are also causative factors. Improper or inadequate surgical technique is assumed to be the cause in most cases of dehiscence, because the use of properly placed retention sutures greatly reduces the incidence of dehiscence. In adults, improper closure of fascia around an ostomy or the inclusion of a stoma within an incision also predisposes to dehiscence.
Wound dehiscence is also associated with vitamin deficiency, drug treatment, and other therapy. Vitamin C deficiency is associated with wound disruption, particularly in elderly patients. Zinc, which is an important cofactor for epithelial and fibroblast growth and function, may be deficient due to malnutrition, chronic diarrhea, or hepatic disease. Chronic steroid use, chemotherapy, and radiation therapy also impair the inflammatory response of healing and are associated with greater rates of dehiscence.
- Sabiston

Complications of cough include wound dehiscence.
- Cough: A Worldwide Problem. Otolaryngologic Clinics of North America - Volume 43, Issue 1 (February 2010)

How well did you know this?
1
Not at all
2
3
4
5
Perfectly
2
Q

Which of the following is the most ideal place to bring out an end colostomy from a sigmoid colon resection:

a) left upper quadrant lateral to the rectus sheath
b) Right lower quadrant above the inguinal ligament
c) Left lower quadrant through the rectus sheath
d) Left lower quadrant lateral to the rectus sheath

A

c) Left lower quadrant through the rectus sheath

  • left b/c sigmoid colon
  • through the rectus sheath to help it stay together
How well did you know this?
1
Not at all
2
3
4
5
Perfectly
3
Q

A patient develops acute pancreatitis post ERCP. The best way to feed this patient is:

a) Enteral
b) parenteral
c) enteral via gastric tube
d) clear fluids per os

A

a) Enteral

Three randomized control trials have demonstrated that enteral feeding is not only safe and feasible but also associated with fewer infectious complications and lower cost than parenteral nutrition is.
- Goldman: Cecil Medicine, 23rd ed.

Several investigative groups have recently demonstrated that most patients with pancreatitis, including those with severe pancreatitis, can actually tolerate small amounts of enterally administered nutrients. They have shown that those nutrients can be tolerated if given either into the stomach (through a nasogastric tube) or into the small intestine (through a nasojejunal tube).
- Townsend: Sabiston Textbook of Surgery, 18th ed.

How well did you know this?
1
Not at all
2
3
4
5
Perfectly
4
Q
What is the most common abnormality associated with a Meckel’s diverticulum?
A. Patent urachus
B. Enteroumbilical fistula 
C. Ectopic gastric mucosa
D. Intestinal volvulus
A

C. Ectopic gastric mucosa
most common complication = gi bleeding

The Meckel diverticulum occurs as a result of failed involution of the vitelline duct, which connects the lumen of the developing gut to the yolk sac. This solitary diverticulum is a small pouch extending from the antimesenteric side of the bowel ( Fig. 17-2 ). It is a true diverticulum with a wall that includes mucosa, submucosa, and muscularis propria. Meckel diverticulae occur in approximately 2% of the population, are generally present within 2 feet (85 cm) of the ileocecal valve, are approximately 2 inches (5 cm) long, are twice as common in males as in females, and are most often symptomatic by age 2 (although only ∼4% of Meckel diverticulae are symptomatic). These facts comprise the “rule of 2s” that is often used to help remember characteristics of Meckel diverticulae. The mucosal lining of Meckel diverticulae may resemble that of normal small intestine, but ectopic pancreatic or gastric tissue may also be present.
- Kumar: Robbins and Cotran Pathologic Basis of Disease, Professional Edition , 8th ed.

Intestinal obstruction caused by intussusception, volvulus, herniation, or entrapment of a loop of bowel through a defect in the diverticular mesentery (6%)
The majority of complicated cases of Meckel diverticulum contain ectopic mucosa (75% gastric, 15% pancreatic).
- Ferri: Ferri’s Clinical Advisor 2010, 1st ed.

How well did you know this?
1
Not at all
2
3
4
5
Perfectly
5
Q

Cancer of the head of the pancreas obstructing the bile duct will cause all of the following except:

a) Increased serum bilirubin
b) Increased urine bilirubin
c) Increased urine urobilirubin
d) Decreased stool pigmentation

A

Urine urobilinogen is decreased or absent when normal amounts of bilirubin are not excreted into the intestinal tract. This usually indicates partial or complete obstruction of the bile ducts.

How well did you know this?
1
Not at all
2
3
4
5
Perfectly
6
Q

What is the most common cause of appendicitis?

a) Fecalith
b) Immunologic tissue hyperplasia
c) Infection
d) Crohn’s

A

DEPENDS ON THE AGE??
a) Fecalith

Answer = fecalith vs immunolgoic tissue hyperplasia

obstruction related to lymphoid hyperplasia (>60%)
- ? source

In the young, lymphoid follicular hyperplasia due to infection is thought to be the main cause. In older patients, luminal obstruction is more likely to be caused by fibrosis, fecaliths, or neoplasia (carcinoid, adenocarcinoma, or mucocele)
-uptodate

Acute appendicitis is thought to be initiated by progressive increases in intraluminal pressure that compromise venous outflow. In 50% to 80% of cases, acute appendicitis is associated with overt luminal obstruction, usually caused by a small stone-like mass of stool, or fecalith, or, less commonly, a gallstone, tumor, or mass of worms (oxyuriasis vermicularis).
- Kumar: Robbins and Cotran Pathologic Basis of Disease, Professional Edition , 8th ed.

Review of pathologic series shows that luminal obstruction is found in the minority of cases. Fecaliths are present in only 8% to 44% of cases of acute appendicitis, with most series at the lower end of the range, [14] [15] [17] and lymphoid hyperplasia is more common in noninflamed appendices than in acute appendicitis.
- Feldman: Sleisenger & Fordtran’s Gastrointestinal and Liver Disease, 8th ed.

How well did you know this?
1
Not at all
2
3
4
5
Perfectly
7
Q

What is the difference between an omphalocele and gastrocshisis?

A

Omphalocele has the sac and gastrocshisis has no sac at all

Omphalocele- the intestines or other body organs extend outside the infant’s abdominal cavity through an opening in the area of the umbilical cord. The protruding intestines or organs are covered with a thin-like membrane.

Gastroschisis- there is also an opening in the abdominal wall through which internal organs spill out of the abdominal cavity. The opening develops in the same manner as with an omphalocele, and the opening can also be small or large. However, in newborns with gastroschisis, the aperture is usually located on the right side of the umbilical cord and the organs are not covered with a membrane. Since the organs are uncovered in the amniotic fluid, there is a greater chance of infection or damage to them.

How well did you know this?
1
Not at all
2
3
4
5
Perfectly
8
Q

9 month pregnant lady with RUQ pain. What is the most likely diagnosis:

a. Acute cholecystitis
b. Gastric ulcer
c. GERD
d. Acute Appendicitis
e. PDU

A

Acute appendicitis is the most common nonobstetric surgical emergency during pregnancy, with an incidence of about 1 per 1,000 pregnancies. Displacement of the appendix by the gravid uterus during late pregnancy may cause the point of maximal abdominal pain and tenderness to migrate superiorly and laterally from McBurney’s point.
- Gabbe: Obstetrics: Normal and Problem Pregnancies, 5th ed.

How well did you know this?
1
Not at all
2
3
4
5
Perfectly
9
Q

Duodenal Atresia is most often associated with:

a. Down’s syndrome
b. Cardiac anomalies
c. Trisomy
d. Radiation exposure
e. Marfan syndrome

A

a. Down’s syndrome

Trisomy 21 is strongly associated with duodenal atresia / stenosis / web in that anywhere from 25% to more than 50% of cases occur in infants and children with this chromosomal anomaly.
- Feldman: Sleisenger & Fordtran’s Gastrointestinal and Liver Disease, 8th ed.

Duodenal atresia is complete obliteration of the duedenal lumen. Etiology is unknown… but thought to be due to failure to recanalize 2nd part of duodenum. 1 in 6000 births. 50% of infants with duodenal atresia are born prematurely. Most commonly associated with trisomy 21. About 22-30% of patients with duodenal obstruction have trisomy 21.

How well did you know this?
1
Not at all
2
3
4
5
Perfectly
10
Q

In a complete SBO:

a. The small bowel remains sterile
b. Fecal organism predominate
c. Gram positive organisms predominate
d. Mixed organism predominate
e. Fungi predominate

A

b. Fecal organism predominate?

In the absence of intestinal obstruction, the jejunum and proximal ileum of the human are virtually sterile. With obstruction, however, the flora of the small intestine changes dramatically, in both the type of organism (most commonly Escherichia coli, Streptococcus faecalis, and Klebsiella species) and the quantity, with organisms reaching concentrations of 10^9 to 10^10/mL.
- Townsend: Sabiston Textbook of Surgery, 18th ed.

How well did you know this?
1
Not at all
2
3
4
5
Perfectly
11
Q

What vitamin deficiency will short bowel syndrome have?

A

Vitamin B12 if less than 100 cm of ileum b/c bile salts still absorbed
Resection of over 100 cm of ileum leads to a reduction in the bile salt pool that results in steatorrhea and malabsorption of fat-soluble vitamins (ADEK).
(see detailed notes)

Vitamin K is absorbed predominantly in the ileum and requires the presence of bile salts. Clinically significant vitamin K deficiency occurs with malabsorption of fat-soluble vitamins secondary to obstructive jaundice or with malabsorption caused by intrinsic small bowel diseases, including celiac sprue, short-bowel syndrome, and inflammatory bowel disease.
- Goldman: Cecil Medicine, 23rd ed.

How well did you know this?
1
Not at all
2
3
4
5
Perfectly
12
Q

The most common cause of colovesicular fistula is:

a. Crohn’s disease
b. Diverticulitis
c. Radiation
d. Bladder cancer
e. Colon Cancer

A

b. Diverticulitis

Diverticulitis accounts for approximately 50%-70% of vesicoenteric fistulae, almost all of which are colovesical.
- http://emedicine.medscape.com/article/442000-overview

How well did you know this?
1
Not at all
2
3
4
5
Perfectly
13
Q

30 year old male patient presents to the clinic with bright red blood per rectum. the next step in management is:

a. Full colonoscopy and barium enema
b. Colonoscopy and banding haemorrhoids
c. Sigmoidscopy and banding of haemorrhoids
d. Sigmoidoscopy only
e. Banding of haemorrhoids only

A

Answer: Sigmoidscopy and banding of haemorrhoids

Anoscopy is the definitive examination, but a flexible proctosigmoidoscopy should always be added to exclude proximal inflammation or neoplasia. Colonoscopy or barium enema should be added if the hemorrhoidal disease is unimpressive, the history is somewhat uncharacteristic, or the patient is older than 40 years or has risk factors for colon cancer, such as a family history.
- Townsend: Sabiston Textbook of Surgery, 18th ed

How well did you know this?
1
Not at all
2
3
4
5
Perfectly
14
Q

1) All are manifestation of abdominal compartment syndrome except:
a. UOP 10cc/hr
b. Increase airway pressure
c. Hypoxia
d. Elevated CVP

2) Patient suddenly becomes hypotensive while intubated and ventilated. Which of the following is a sign of abdominal compartment syndrome
a – increased peak inspiratory pressure
b – deceased cardiac output
c – lymphocytosis

A

c?
b- reduced cardiac output

I’m not sure

Clinical manifestations:
Cardiovascular: cephalad movement of diaphragm. Can cause direct cardiac compression, reducing both ventricular compliance and contractility. Functionally obstruct IVC and femoral viens and subsequently reduce venous return to the heart. Invasively measured intravascular pressures, including PCWP and CVP are elevated with increased IAP despite reduced venous return and CO. But… not reflective of true intravascular volume.
Pulmonary: intraabdominal pressure is transmitted through the thorax through the elevated diaphragm, resulting in extrinsic compression of pulm parenchyma. This results in atelectasis, edema, decrease o2 transport, increased intrapulm shunt fraction and increased alveolar dead space. In mechanically ventillated patient peak inspiratory and mean airway pressures are increased.
Renal: decreased renal function caused directly by increased venous resistence (and therefore impaired venous drainage) and indirectly by arterial vasoconstriction mediated by stim. of sympathetic nervous and renin-angiotensin systems by the fall in cardiac output. ** renal dysfunction thought to be more from increased renal venous pressure rather then direct compression on parenchyma
G.I.: changes in mesenteric blood flow as low as 10mmHG. Decreased intestinal mucosal blood flow at 20 mmHg and at 40 mmHg celiac artery blood flow and SMA are decreased by 43 and 69 %
Hepatic: decreased ability of liver to clear lactic acid (only at 10mmHg)
CNS: elevated CVP can induce increases in ICP. This in combo with hypotension and decrease CO can lead to cerebral hypoperfusion.

The findings of a tensely distended abdomen, progressive oliguria despite adequate CO, or hypoxia with increasing airway pressure are sufficient to justify abdominal decompression.
- Townsend: Sabiston Textbook of Surgery, 18th ed.

How well did you know this?
1
Not at all
2
3
4
5
Perfectly
15
Q

70 year old patient with history of A.fib and arthritis. Developed sudden severe abdominal pain. Had one BM today and vomited twice. Abdominal exam revealed generalized tenderness but not peritonitis. She had tubal ligation in the past. What is the most probable diagnosis:

a. Acute mesenteric embolic ischemia
b. Perforated PUD
c. Intestinal obstruction
d. Rupture AAA

A

Answer: Acute mesenteric embolic ischemia

keyword: afib, general tenderness but not peritonitis

If on coumadin and aspirin could have bleeding PUD.
Afib if non anticoagulated could cause emboli.

How well did you know this?
1
Not at all
2
3
4
5
Perfectly
16
Q

a 56 year old female develops abdominal pain and sepsis with an anion gap metabolic acidosis 4 days after undergoing a triple vessel coronary artery bypass graft. She is diagnosed with a small bowel infarction. The most likely cause is

a. ileus
b. superior mesenteric vein thrombosis
c. arterial thrombosis secondary to low flow
d. arterial thrombosis secondary to emboli
e. none of the above

A

Answer: arterial thrombosis secondary to emboli
???
Embolization to the intestine via the SMA (SMA embolus) accounts for 5% of peripheral emboli and nearly 50% of cases of primary noncolonic mesenteric ischemia. Emboli most commonly originate from the heart, with an aortic origin being less common ( Table 146-2 ), and tend to obstruct beyond the origin of the SMA.
- Goldman: Cecil Medicine, 23rd ed.

Acute Mesenteric Ischemia: Metabolic acidosis is a late finding.
- Feldman: Sleisenger & Fordtran’s Gastrointestinal and Liver Disease, 8th ed.

How well did you know this?
1
Not at all
2
3
4
5
Perfectly
17
Q

All of the following are relative contraindications to laparoscopy except

a. abdominal wall infection
b. coagulopathy
c. congestive heart failure
d. umbilical hernia
e. right heart failure (cor pulmonale)

A

d. umbilical hernia

[P]atients who are not candidates for a general anesthetic, such as those with severe cardiopulmonary disease, should not undergo laparoscopy. Physicians must be cognizant of conditions that may alter a patient’s physiology, such as the pneumoperitoneum, which may further compromise ventilation, and compression of the vena cava, which may limit venous return (Arthure, 1970; Hodgson et al, 1970; Nunn, 1987; Lew et al, 1992). Patients with mild to moderate chronic obstructive pulmonary disease may have difficulty compensating for the hypercarbia, and the pneumoperitoneum may need to be kept at lower pressures than usual (Monk and Weldon, 1992; Adams et al, 1995a). Laparoscopy should also be avoided in patients with severely dilated bowels from either functional or obstructive ileus. In these cases, the dilated intestines take up working space and may be injured during access and dissection (Borten, 1986). Other absolute contraindications include uncorrected coagulopathy, untreated infection, and hypovolemic shock (Capelouto and Kavoussi, 1993).
Several conditions require caution when considering a laparoscopic approach. Prior intra-abdominal or retroperitoneal surgery is not a contraindication to laparoscopy; however, each case must be carefully assessed. Prior transperitoneal surgery can cause bowel adhesions to the abdominal wall or scar tissue formation about the operative site, increasing the possibility of injury during insertion of the Veress needle, trocar placement, or dissection (Borten, 1986). In approaching these patients, the Veress needle should be placed away from any scars and any prior surgical fields. Alternatively, open trocar placement can be undertaken to minimize access injuries (Hassan, 1971).
Obese patients should also be approached with discretion because abdominal wall fat may make trocar placement difficult and mask anatomic landmarks (Mendoza et al, 1996). Moreover, the weight of the pannus may raise the intra-abdominal pressure and limit the working space.
- Sabiston

How well did you know this?
1
Not at all
2
3
4
5
Perfectly
18
Q

all can cause post OP complication with known liver disease except :

  • increase bilurubin
  • increase PT
  • AST, ALT
  • Ascitis
  • decrease albumin
A

Answer: ?ALT/AST

Child-Pugh Classification
Albumin, Bilirubin, PTT/INR, ascites, encephalopathy

The Model for End-Stage Liver Disease (MELD) score has replaced the Child-Pugh classification as the predominant prognostic model in end-stage liver disease. [21] [34] The MELD score, initially validated in patients undergoing elective transjugular intrahepatic portosystemic shunting, uses serum creatinine, bilirubin, and international normalized ratio. It is an independent predictor of mortality in patients with cirrhosis or those who are awaiting liver transplantation.

MODIFIED CHILD-PUGH CLASSIFICATION AND THE MELD SURVIVAL MODEL
			Numerical Score
Parameter		1	2	3
Ascites			None	Slight	Moderate or severe
Encephalopathy	None	Gr 1-2	Gr 3-4
Bilirubin (mg/dl)	 3
Albumin (mg/L)	> 3.5	2.8-3.5	 6
 - Piccini & Nilsson: The Osler Medical Handbook, 2nd ed.
How well did you know this?
1
Not at all
2
3
4
5
Perfectly
19
Q

Strongest contraindication to PEG tube

a. history of colon resection
b. previous lap chole
c. inability to transilluminate abdominal wall
d. previous appendectomy
e. reflux esophagitis

A

Answer: inability to transilluminate abdominal wall No longer a contra-indication****

Absolute contraindications
Inability to perform an esophagogastroduodenoscopy
Uncorrected coagulopathy
Peritonitis
Untreatable (loculated) massive ascites
Bowel obstruction (unless the PEG is sited to provide drainage)

Relative contraindications
Massive ascites
Gastric mucosal abnormalities: large gastric varices, portal hypertensive gastropathy
Previous abdominal surgery, including previous partial gastrectomy: increased risk of organs interposed between gastric wall and abdominal wall
Morbid obesity: difficulties in locating stomach position by digital indentation of stomach and transillumination
Gastric wall neoplasm
Abdominal wall infection: increased risk of infection of PEG site
- Gastroenterological endoscopy. Meinhard Classen, G. N. J. Tytgat, Charles J. Lightdale. 2002

A number of contraindications to PEG tube placement exist:

  1. No endoscopic access
  2. Severe coagulopathy
  3. Gastric outlet obstruction
  4. Anticipated survival of less than 4 weeks
  5. Inability to approximate the gastric wall to the abdominal wall
    - Townsend: Sabiston Textbook of Surgery, 18th ed.
How well did you know this?
1
Not at all
2
3
4
5
Perfectly
20
Q

A 30 year old male has a palpable node along the right inguinal ligament. What WOULD NOT be a possible source?

a) cellulitis of the right thigh
b) a scrotal incision
c) a lesion of the glans of the penis
d) a testicular tumour
e) a mass on the lower right abdominal wall

A

Answer: a testicular tumour – drains to para-aortic nodes

Testicular cancer [has] a very predictable and systematic pattern of metastatic spread from the primary site to the retroperitoneal lymph nodes and, subsequently, to the lung and posterior mediastinum
- Wein: Campbell-Walsh Urology, 9th ed.

How well did you know this?
1
Not at all
2
3
4
5
Perfectly
21
Q

The following statements regarding the spleen are true EXCEPT:

a. Lack of Howell-Jolly bodies after splenectomy suggests an accessory spleen or splenosis.
b. Levels of properdin and tuftsin fall after splenectomy.
c. Accessory spleens are found in 15-30% of people.
d. The spleen can only remove cells coated with immunoglobulin A (IgA).
e. All of the above statements are true.

A

Answer: The spleen can only remove cells coated with immunoglobulin A (IgA).

It has also been found that immunoglobulin (Ig) G-sensitized red cells are rapidly removed from the circulation by the spleen and that unusually rapid clearance persists well into the convalescent phase.
- Hoffman: Hematology: Basic Principles and Practice, 5th ed.

How well did you know this?
1
Not at all
2
3
4
5
Perfectly
22
Q

Which of the following is MOST effective in preventing LATE complications of splenectomy?
A. Adminstration of pneumococcal and H. flu vaccines
B. Perform only laparoscopic splenectomy
C. Perform sub-total splenectomy
D. Give patients antibiotic prophylaxis post-operatively

A

Answer: Adminstration of pneumococcal and H. flu vaccines

The current recommendations for patients who are undergoing elective splenectomy include vaccination of persons susceptible to Pneumococcus strains.
For patients who are at particularly high risk because of immunosuppression, polyvalent vaccines are also available against Neisseria meningitidis and Haemophilus influenzae type B

The pneumococcus is singularly the most important organism in PSS, involved in 50% to 90% of cases.[26,27] In a review of 349 PSS cases, Streptococcus pneumoniae was causative in 66% of episodes in which a bacterium could be identified.
Type b H. influenzae (Hib) is the second most common organism in PSS.[26,27] Most Hib-associated PSS cases have occurred in children younger than 15 years, 86% in one review,[27] with a frequency about 10 times lower than that of the pneumococcus. Neither nontypable nor non-b capsular strains (a, c-f) are significant PSS pathogens. Use of the conjugated Hib vaccine has dramatically decreased the incidence of invasive Hib disease.
- Mandell: Mandell, Douglas, and Bennett’s Principles and Practice of Infectious Diseases, 7th ed.

Pneumococcal infections account for 50-90% of cases reported in the literature and may be associated with a mortality rate of up to 60%. H influenza type B, meningococci, and group A streptococci account for an additional 25% of infections

How well did you know this?
1
Not at all
2
3
4
5
Perfectly
23
Q
Which of the following is indicative of a blood test post splenectomy?
A. Thrombocytosis
B. Neutropenia
C. Spherocytosis
D. Anemia
A

A. Thrombocytosis
Can also have leukocytosis

“Leukocytosis and increased platelet counts commonly occur following splenectomy as well. “

Postsplenectomy thrombocytosis may be associated with both hemorrhagic and thromboembolic phenomena.
- Townsend: Sabiston Textbook of Surgery, 18th ed.

Granulocytosis immediately, then Thrombocytosis, RBC’s have inclusion bodies ie Howell-Jolly bodies and friends…

Splenectomy results in characteristic changes to blood composition, including the appearance of Howell-Jolly bodies and siderocytes. Leukocytosis and increased platelet counts commonly occur following splenectomy as well.

In addition, thrombocytosis and leukocytosis are observed because the spleen functions as a reservoir for these blood cells.
- http://emedicine.medscape.com/article/885226-diagnosis

How well did you know this?
1
Not at all
2
3
4
5
Perfectly
24
Q

All of the following have an enlarged spleen except:

a) ITP
b) Liver cirrhosis

A

ITP.

Splenomegaly excludes the diagnosis of ITP!

How well did you know this?
1
Not at all
2
3
4
5
Perfectly
25
Q

With regards to intra-aortic balloon pumps, which of the following is TRUE?
A. It consistently decreases inotropic dosages required
B. It deflates during early systole
C. It maintains a high diastolic pressure
D. It increases cardiac preload

A

C. It maintains a high diastolic pressure

Intraaortic balloon pump (IABP) counterpulsation is a very effective means of increasing diastolic coronary blood flow and reducing left ventricular afterload, which act in concert to reduce ischemia.
- Libby: Braunwald’s Heart Disease: A Textbook of Cardiovascular Medicine, 8th ed.

As it is inflated with an internal pressure of helium that is greater than intraaortic pressure, it will displace 40ml of blood from the aorta. This added volume causes an increase in aortic pressure and diastolic blood flow (diastolic augmentation). The rapid deflation of the balloon leaves a 40ml void, which effectively decreases aortic pressure and thus left ventricular afterload, during early systole. These beneficial hemodynamic changes – diastolic augmentation and systolic afterload reduction – can greatly increase coronary blood flow and decrease myocardial oxygen consumption.
- Critical care handbook of Mass General pg 219

Intra-aortic balloon pumping improves mean blood pressures and coronary artery perfusion and decreases cardiac work and oxygen demand.
- Townsend: Sabiston Textbook of Surgery, 18th ed.

Insertion of an intra-aortic balloon pump (IABP) for counterpulsation increases diastolic coronary artery perfusion pressure, decreases LV afterload, improves cardiac output, and decreases myocardial oxygen demand.
Management of RV infarction consists of early maintenance of RV preload with intravenous fluids, reduction of RV afterload (i.e., afterload-only reducing drugs as for LV dysfunction; consider intra-aortic balloon pump).
- Goldman: Cecil Medicine, 23rd ed.

balloon deflated prior to and during early left ventricular ejection thus reducing aortic pressure and thus afterload.
- http://www.aic.cuhk.edu.hk/web8/IABP.htm

How well did you know this?
1
Not at all
2
3
4
5
Perfectly
26
Q

Which of the following is a contraindication for Intra-Aortic Balloon Pump?

a) severe AS
b) severe AR
c) CHF
d) Recent MI
e) ventricular aneurysm

A

The use of IABP is absolutely contraindicated in the setting of aortic insufficiency and aortic dissection. Relative contraindications for use via a femoral arterial approach relate to the vascular consequences of cannulation—significant aortoiliac or iliofemoral disease, and abdominal or descending thoracic aortic aneurysms.
- Libby: Braunwald’s Heart Disease: A Textbook of Cardiovascular Medicine, 8th ed.

How well did you know this?
1
Not at all
2
3
4
5
Perfectly
27
Q

Anterior mediastinal mass, is likely
a-Lymphoma
b-Lung Cancer
c-Thymoma was not one of the options

A

a-Lymphoma

How well did you know this?
1
Not at all
2
3
4
5
Perfectly
28
Q

Most common anterior mediastinal mass is:

a) Lymphoma
b) Teratoma
c) Bronchogenic cyst
d) Pericardial cyst

A

1) Lymphoma

All mediastinal masses:
Thymoma (20%) > Lymphoma > Teratoma or Thyroid goiter

Anterior mediastinum
Four Ts
Thymoma (45%)
“Terrible” lymphoma (20%)
Teratoma (15%)
Thyroid (or parathyroid) (15%)
Also neurogenic or mesenchymal tumors (5%)

4 T’s” (Thymoma, Terrible lymphoma, Teratoma, and Thyroid)

Middle Mediastinum:

  • Lymphoma
  • bronchogenic lung CA
Posterior Mediastinum  
neurogenic tumors (schwannoma, neurofibroma,)
How well did you know this?
1
Not at all
2
3
4
5
Perfectly
29
Q
A patient presents with fatigue, muscle weakness and tires with chewing. A CXR reveals a 7cm mass in the anterior mediastinum. Which of the following tests would CONFIRM the diagnosis?
A. CT with contrast
B. Tensilon test  
C. Deltoid muscle biopsy
D. Bronchoscopy
A

B. Tensilon test
About 50% of patients with an underlying thymic tumour are asymptomatic, and diagnosis is often incidental. However, the most important factor to elicit is a history of myasthenia gravis or muscle weakness consistent with myasthenia gravis (about 30% of patients with a thymoma have associated myasthenia gravis). Patients with myasthenia gravis may have ocular symptoms only (double vision, drooping eyelids) or generalised muscle weakness, especially facial and oropharyngeal weakness (suggested by difficulty in chewing and swallowing) and/or proximal limb weakness (suggested by difficulty with getting out of chairs or climbing stairs).

The diagnosis of MG is based on clinical disease history and typical clinical findings. MG can be confirmed pharmacologically by edrophonium (Tensilon) test which is positive in 90% of MG patients, giving an immediate but transitory improvement of MG signs.

A battery of tests can be used in the assessment of the patient with suspected myasthenia gravis, but only a few are available to the emergency clinician. ACh receptor antibody assay is positive in more than 80% of patients with myasthenia but has a turnaround time of several days. Repetitive nerve stimulation and single-fiber electromyography tests are available in the electrophysiology laboratory. Several pharmacologic tests have been used to aid in the diagnosis of suspected myasthenia gravis including parenteral administration of edrophonium chloride (Tensilon), neostigmine, or curare. Edrophonium chloride administration for diagnosis of myasthenia gravis (Tensilon test) is described in detail because of the drug’s rapid onset, short duration of action, and widespread acceptance for this diagnostic challenge.
- Roberts: Clinical Procedures in Emergency Medicine, 5th ed.

How well did you know this?
1
Not at all
2
3
4
5
Perfectly
30
Q

All the following are absolute indications for thoracotomy tube except:

a) open pneumothorax
b) spontaneous pneumothorax
c) empyema
d) chyleme

A

b) spontaneous pneumothorax

Clinically stable patients with small pneumothoraces (PTXs) should be observed in the emergency department (ED) for 3–6 hr and discharged home if a repeat chest radiograph excludes progression of the PTX (good consensus).
- Roberts: Clinical Procedures in Emergency Medicine, 5th ed.

How well did you know this?
1
Not at all
2
3
4
5
Perfectly
31
Q

A man is injured waterskiing, resulting in a massive laceration across his chest. He presents to the ER with a stable BP of 100/60, HR of 100, and a chest tube is inserted draining 1200 ml immediately. He is stabilized at the community hospital and when he arrives at the trauma center he continues to drain 600 ml over the hour. What is the next step?

a) CT chest
b) Insert another chest tube
c) Irrigate, debride and repair the chest wound and clamp off the chest tube
d) Emergent thoracotomy

A

d) Emergent thoracotomy

Indications for emergency thoracotomy are initial chest tube output of 1500 mL of blood or persistent drainage of 200 to 300 mL/hr.
- Townsend: Sabiston Textbook of Surgery, 18th ed.

How well did you know this?
1
Not at all
2
3
4
5
Perfectly
32
Q
A chest xray shows a right tension pneumothorax and a left diaphragmatic hernia. What is the MOST appropriate immediate action?
A. Insert chest tube on the right
B. Insert chest tube on the left
C. Perform an emergent thoracotomy
D. Insert NG tube
A

A. Insert chest tube on the right

How well did you know this?
1
Not at all
2
3
4
5
Perfectly
33
Q

The commonest cause of empyema is:

a. Malignancy
b. Bacterial pneumonia
c. Asthma
d. Viral pneumonia
e. Trauma

A

b. Bacterial pneumonia

Infections of the pleural space most commonly follow pneumonia, accounting for 40% to 60% of all empyemas. Thoracotomy is the next most common precursor of empyema, accounting for 20%, and trauma accounts for 4% to 10%. Less commonly, empyema can develop as a result of esophageal rupture and subdiaphragmatic spread. Other uncommon causes include hematogenous seeding of an existing pleural effusion and direct extension from head and neck infections.
- Mandell: Mandell, Douglas, and Bennett’s Principles and Practice of Infectious Diseases, 7th ed.

CAUSES OF EMPYEMA 
Cause						Frequency (%)
Pyogenic pneumonia				50
Postsurgical					25
Subphrenic abscess extension			10
Posttraumatic					3–5
Lung abscess rupture				1–3
Generalized sepsis				1–3
Pulmonary tuberculosis			1
Pulmonary mycotic infection			1
Spontaneous pneumothorax
How well did you know this?
1
Not at all
2
3
4
5
Perfectly
34
Q

The best management of chronic empyema:

a. Decortication
b. Pneumonectomy
c. IV antibiotics
d. Lobectomy
e. Chest tube insertion

A

Answer: Decortication

Phases: Proliferative, Exudative, Fibrino-purulent, Fibrosis

How well did you know this?
1
Not at all
2
3
4
5
Perfectly
35
Q
Which of the following is the MOST common cause of a spontaneous pneumothorax?
A. Cystic Fibrosis
B. Pulmonary Abscess
C. Apical Bleb
D. Emphysema
A

C. Apical Bleb
vs. emphysema

The question is awkward because it doesn’t consider primary vs secondary. But … it may be easier to answer based on the scenario (young skinny tall guy vs. old smoker)

The incidence of secondary spontaneous pneumothorax is similar to that of primary spontaneous pneumothorax.
- Mason: Murray & Nadel’s Textbook of Respiratory Medicine, 4th ed.

Blebs can be found in more than 75% of patients undergoing thoracoscopy for treatment of primary spontaneous pneumothorax.
- Mason: Murray & Nadel’s Textbook of Respiratory Medicine, 4th ed.

The most common condition associated with secondary spontaneous pneumothorax is chronic obstructive pulmonary disease (COPD), which accounts for nearly 70% of cases.
- Marx: Rosen’s Emergency Medicine, 7th ed.

So maybe bleb is the right answer? The answer key says emphysema.

Spontaneous pneumothorax is a collection of air or gas in the chest that causes the lung to collapse in the absence of a traumatic injury to the chest or lung. A primary spontaneous pneumothorax usually occurs at rest.

Spontaneous pneumothorax is seven times more likely to occur in males than females. Male smokers have 22 times the likelihood of spontaneous pneumothorax compared to non-smoking males. Female smokers have a nine times increase in the likelihood of a spontaneous pneumothorax compared to nonsmoking females. A spontaneous pneumothorax is most likely to occur during the fall or winter months. There are between 800 and 900 cases a year in Canada.

Usually, the rupture of a small bleb or bullae (an air- or fluid-filled sac in the lung) causes primary spontaneous pneumothorax. Secondary spontaneous pneumothorax occurs in the setting of known lung disease, most often chronic obstructive pulmonary disease (COPD). Other lung diseases commonly associated with spontaneous pneumothorax include: tuberculosis, pneumonia, asthma, cystic fibrosis, lung cancer and certain forms of interstitial lung disease.

If left untreated, recurrence rates of a spontaneous pneumothorax are high. Without treatment same side recurrence rates are as high as 30% at six months and up to 50% at two years.

Treatment of spontaneous pneumothorax
The objective of treatment is to remove the air from the pleural space, allowing the lung to re-expand. Small pneumothoraces may resolve on their own.

Aspiration of air, through a catheter to a vacuum bottle, may re-expand the lung.

The placement of a chest tube between the ribs into the pleural space allows the evacuation of air from the pleural space when simple aspiration is not successful or when the pneumothorax is large. Re-expansion of the lung may take several days with the chest tube left in place. Hospitalization is required for chest tube management.

Surgery may be recommended for recurrent episodes.

How well did you know this?
1
Not at all
2
3
4
5
Perfectly
36
Q

A 23 year old male presents with a second episode of a spontaneous pneumothorax. He is otherwise healthy. His management includes chest tube drainage ad which ONE of the following?

a) Apical bullectomy (via thoracoscopy)
b) CT chest
c) Chest tube drainage only
d) pleuradhesis

A

a) Apical bullectomy (via thoracoscopy)

We suggest that patients who are clinically stable with a recurrent PSP undergo chest tube insertion followed by VATS Video-assisted thoracoscopic surgery

Following a primary spontaneous pneumothorax, a patient is at risk of recurrence particularly in the months immediately after the first episode. Sadikot and associates[13] followed up 153 patients with primary spontaneous pneumothorax for a mean of 54 months and reported that 39% had a recurrent ipsilateral pneumothorax, most within the first year. Once a patient has a primary spontaneous pneumothorax there is also an increased risk of having a pneumothorax on the contralateral side. In the report of Sadikot and coworkers,[13] 15% of their 153 patients developed a pneumothorax on the contralateral side as well.
There have been several attempts to predict who will develop recurrent pneumothorax. Patients who are tall and those who continue to smoke are more likely to have a recurrence.[13] Yet, there is no relationship between the number of blebs or the size of the blebs on computed tomography (CT)[14] or the appearance of the lung at thoracotomy[15] and the risk of recurrence. Once a patient has had one recurrence, the risk of another recurrence increases to more than 50%.[16]
Most patients with their first primary spontaneous pneumothorax should be managed initially with simple aspiration of the pleural air with a plastic catheter. If this is ineffective, thoracoscopy should be performed promptly, with endo-stapling of blebs and pleural abrasion to create a pleurodesis. Patients with recurrent primary spontaneous pneumothorax should undergo thoracoscopy. When thoracoscopy is unavailable, patients who fail aspiration should be managed with tube thoracostomy with an attempt at pleurodesis with either doxycycline or talc. If, after 3 days of thoracostomy drainage, there is a persistent air leak or the lungs have not re-expanded, thoracotomy should be considered.
- Murray & Nadel: Textbook of Respiratory Medicine, 3rd ed.

Surgical treatment (usually thoracotomy with abrasion of pleural surfaces) is advocated at the time of the first or second recurrence. [33] [34] Patients who have had one pneumothorax have a 30 to 50% chance of ipsilateral recurrence within 1 to 2 years. [33] After a second spontaneous pneumothorax, the probability of a third rises to 50 to 80%. [33] [35] Surgery may be recommended on the occasion of a patient's first pneumothorax in a number of situations: life-threatening tension pneumothorax, massive air leaks with incomplete reexpansion, an air leak persisting 4 to 5 days after a second intercostal tube has been placed, associated hemothorax with complications, cases of identifiable bullous disease, and failure of easy reexpansion in patients with cystic fibrosis.  
 - Roberts: Clinical Procedures in Emergency Medicine, 3rd ed., Copyright © 1998
How well did you know this?
1
Not at all
2
3
4
5
Perfectly
37
Q

A 70 year old ♂ with a history of COPD presents acutely SOB after a vomiting episode. He is diagnosed with a right pneumothorax on presentation. He was appropriately treated with right-sided chest tube and underwater closed suction. 48 hours later, he becomes febrile, with chills, elevated WBCs, and brown purulent, foul-smelling drainage from chest tube. The MOST likely cause is:
A. Empyema secondary to insertion of chest tube
B. Esophageal perforation
C. Necrotizing pneumonia
D. Infected Hematoma

A

The diagnosis of esophageal rupture should always be considered in the differential diagnosis of exudative pleural effusions because, if this condition is not rapidly treated, the mortality increases, with current estimates ranging between 30% and 60%.[294] Approximately 60% of patients with esophageal perforation have a pleural effusion, whereas about 25% have a pneumothorax.[295] The pleural effusion is usually left sided ( Fig. 68.9 ), but it may be right sided or bilateral.
- Mason: Murray & Nadel’s Textbook of Respiratory Medicine, 4th ed

How well did you know this?
1
Not at all
2
3
4
5
Perfectly
38
Q

Older female with episode of vomiting resulting in severe retrosternal chest pain, SQ emphysema and a tender neck. CXR done, what is the next step:

a) CT chest
b) Contrast esophagogram
c) Esophagoscopy – CI
d) Bronchoscopy

A

b) Contrast esophagogram
2016: uptodate says Dx with contrast esophagogram or CT

When the diagnosis is considered, a water-soluble contrast agent should be administered. If this study is negative, dilute barium should be administered. Barium is relatively inert, and the fear of barium extravasating into the mediastinum through the site of injury and producing a severe reactive mediastinitis is unfounded. The risk of barium leaking into the mediastinum is far less than that of failing to recognize the perforation in a timely fashion. Also, because barium provides far better mucosal detail than water-soluble agents, only if barium has been used for the esophagogram should this study in search of a perforation be termed negative.
- Greenfield
Esophagogram is a contrast radio-graph of the esophagus. Gastrografin Contrast Esophagogram is often used to confirm the diagnosis and to locate the position of esophagial perforation.

How well did you know this?
1
Not at all
2
3
4
5
Perfectly
39
Q

Following gastroscopy with esophageal dilatation, a patient develops retrosternal chest pain. EKG shows tachycardia with no other abnormality. A cxr shows air within the mediastinum. A gastrograffin swallow shows extravasation of contrast. The next treatment is:

a. observation
b. antibiotics and ng tube placement fluoroscopically
c. antibiotics and repeat gastroscopy with NG tube placement
d. urgent thoracotomy and primary esophageal repair
e. none of the above

A

e? do a esophagogram or CT

Nonoperative management should be reserved for clinically stable patients with no evidence of systemic inflammation, expediently diagnosed perforations, and no drainage of any collection into the pleura or peritoneum.

A major leakage, or one that occurred through a tumor or an ulcer, requires urgent operation. Limited or absent extravasation on radiographic study allows treatment by intravenous antibiotics and close observation.

The esophagus or stomach is perforated during diagnostic upper endoscopy in 0.03% to 0.1% of procedures. [6] [22] [65] Perforation occurs most commonly when the endoscope is passed blindly–namely, in the pharynx or upper esophagus–and the esophagus is perforated more commonly than the stomach. [37] Osteoarthritic spurs on the cervical spine, Zenker’s diverticulum, anastomoses, strictures, and weakening of the wall of the esophagus or stomach from inflammation, neoplasm, or ingestion of caustic substances are predisposing factors. Perforation of the midesophagus usually results from biopsy of a carcinoma
Endoscopic perforation of the esophagus or stomach may be obvious immediately or within a few hours. Cervical pain, subcutaneous emphysema, fever, tachycardia, and characteristic radiographic appearances make the diagnosis easy, but some distal esophageal injuries are more subtle. [37] An esophagogram should be obtained immediately if perforation is suspected. After ensuring that the patient does not aspirate on swallowing, a water-soluble contrast material should be used for the esophagogram. If the perforation site cannot be identified by this method, a small amount of barium can be used. A major leakage, or one that occurred through a tumor or an ulcer, requires urgent operation. Limited or absent extravasation on radiographic study allows treatment by intravenous antibiotics and close observation.
- Feldman: Sleisenger & Fordtran’s Gastrointestinal and Liver Disease, 6th ed., Copyright © 1998

How well did you know this?
1
Not at all
2
3
4
5
Perfectly
40
Q

The following is due to an abnormality in the cricopharyngeous muscle:

a. Zenkers diverticula
b. Schatzki ‘ s ring
c. Esophageal web

A

a. Zenkers diverticula

Not a true diverticulum (unlike meckle’s)
True diverticulum of the esophagus at the level of mediastium (due to maliganacy)

How well did you know this?
1
Not at all
2
3
4
5
Perfectly
41
Q

SCC of esophageal cancer is not associated with:

a. Barrett’s
b. Achalasia
c. Upper esophageal web
d. Smoking

A

a. Barrett’s (adenocarcinoma)

Squamous cell carcinomas arise from the squamous mucosa that is native to the esophagus and is found in the upper and middle third of the esophagus 70% of the time. This type of cancer is due to exposure to environmental factors. Smoking and alcohol both increase the risk for foregut cancers by 5-fold. Combined, the risk increases from 25- to 100-fold. Food additives, including nitrosamines found in pickled and smoked foods, long-term ingestion of hot liquids, and vitamin (vitamin A) and mineral deficiencies (zinc and molybdenum) have been implicated. Other disorders that expose the esophagus to mucosal trauma including caustic ingestion, achalasia, bulimia, tylosis (an inherited autosomal dominant trait), Plummer-Vinson syndrome, external-beam radiation, and esophageal diverticula all have known associations with squamous cell cancer.
Progressive changes from metaplastic (Barrett’s esophagus) to dysplastic cells may lead to the development of esophageal adenocarcinoma.
- Townsend: Sabiston Textbook of Surgery, 18th ed.

How well did you know this?
1
Not at all
2
3
4
5
Perfectly
42
Q

Risk of transforming Barrett’s esophagus to Adeno ca is :

a. 1% / year
b. 5% /year
c. 10% /year
d. 15% /year

A

Prospectively following 100 patients with Barrett’s esophagus for 1 year will result in 1 patient developing adenocarcinoma, a rate of 1% per year.[13] This is a similar risk to that of patients with a 20-pack-year smoking history developing lung cancer
- Townsend: Sabiston Textbook of Surgery, 18th ed.

How well did you know this?
1
Not at all
2
3
4
5
Perfectly
43
Q

All of the following are complications of chylothorax except :

a. B cell leucopenia
b. Dehydration
c. Electrolytes abnormalities
d. Hypoproteniemia
e. Low glyecero

A

a. B cell leucopenia

i don’t know what a glyecero is

The primary cellular component of chyle is the T lymphocyte. Prolonged loss of chyle results in severe nutritional depletion, water and electrolyte loss, hypolipemia, and lymphopenia of T cells with resulting immunosuppression.
- Murray & Nadel: Textbook of Respiratory Medicine, 3rd ed

Normal chyle flow ranges between 1.5 and 2.5 L/day but can vary much more widely depending on diet and on the fat content of the diet. During starvation or intravenous feeding, flow falls to about 250 mL/day of clear fluid. Chylothorax is frequently massive (Fig. 16-31) and symptomatic, and significant volume losses can occur through thoracentesis or chest tube drainage. Dehydration, nutritional losses, and a steady decline in circulating lymphocytes can produce significant disability and an increased susceptibility to infection
- Schwartz Chap 16

How well did you know this?
1
Not at all
2
3
4
5
Perfectly
44
Q

During neck surgery thoracic duct was injured and the injury was discovered during the surgery the treatment of choice is:

a. Ligate the duct
b. Insert a drain
c. Do nothing
d. Post op antibiotics
e. All of the above

A

a. Ligate the duct

Ligation is 1st option, if you can’t then apply a Drain and start on Medium Chain Fatty Acid Diet to decrease Lymph production, this will slow the output down and may close spontaneously. If not then go back and operate to find and ligate it.

For patients with thoracic duct injury and an effusion that persists after 1 week of drainage, video-assisted thoracoscopy or thoracotomy is employed to identify and ligate the thoracic duct above and below the leak.
- Townsend: Sabiston Textbook of Surgery, 18th ed.

During dissection in the region of the lower jugular vein, the thoracic or lymphatic duct should be carefully ligated if identified.
- Cummings: Otolaryngology: Head & Neck Surgery, 4th ed.

In a recent review of 823 neck dissections performed by the surgeons at Memorial Hospital in New York City that included the removal of the lymph nodes in level IV, Spiro, Spiro, and Strong[68] found that 14 patients (1.9%) developed a chyle fistula. In this and other studies,[17] a chylous leakage was identified and apparently controlled intraoperatively in the majority of patients who developed the complication. These observations remind the surgeon to not only avoid injury to the thoracic duct proper but also to ligate or clip any visualized or potential lymphatic tributaries in the area of the thoracic duct.
- Cummings: Otolaryngology: Head & Neck Surgery, 4th ed.

How well did you know this?
1
Not at all
2
3
4
5
Perfectly
45
Q

A chyle leak found POD 3 from a woman with neck dissection. How to best manage?

a) Leave in closed suction drain and start TPN
b) Take out drain and start TPN
c) Explore and repair duct
d) Administer enteral feeds with medium branched chain fatty acids
e) Leave in drain and make patient NPO

A

Answer:

  • Leave in closed suction drain and start TPN
  • Thoracostomy tube and TPN nutrition

Basically if you’re right there staring at the duct that is leaking, ligate it. If it’s not intra-op, TPN and drain

Total parenteral nutrition allows for control of the fluid and protein loss while avoiding flow of chyle, and in most cases it results in resolution.
Management of chyle fistulization in association with neck dissection.
- Gregor RT - Otolaryngol Head Neck Surg - 01-Mar-2000; 122(3): 434-9

Always consider conservative management because the thoracic duct leak closes spontaneously in nearly 50% of patients. Few or no symptoms and minimal chyle loss characterize these cases.
• Decompress the pleural space with tube thoracostomy or repeated thoracentesis to keep the lung expanded against the chest wall and mediastinum.
• Reduce chyle production by instituting total parenteral nutrition or a fat-restricted oral diet supplemented with medium-chain triglycerides.

How well did you know this?
1
Not at all
2
3
4
5
Perfectly
46
Q

Patient is post-op right thoracotomy and is draining milky fluid via incision. What is the best initial treatment?

  • exploration and ligation of thoracic duct
  • chest tube and TPN
  • NPO
A

Answer: chest tube and TPN

**depends on how much per day. If 500ml/day, for 1st week, then surgical intervention is indicated*****

If significant chyle drainage (>500 mL/d in an adult, >100 mL/d in an infant) continues despite TPN and good lung expansion, early surgical ligation of the duct is recommended.

Treatment of postoperative chylothorax
Postoperative chylothorax represents a grave complication with a mortality rate that may approach 50 percent. Management includes chest tube drainage and parenteral nutrition with the patient fasting or orally supplemented with MCT (enter portal vein system directly. Somatostatin and octreotide have been reported to decrease chyle production in postoperative chylothorax and in dog models with surgical transection of the thoracic duct. Duration of conservative therapy:
1) early repair if young and low risk
2) if drain >1L/day, repair day 5-7
3) if malignancy, repair earlier as malvourishment and immunocomprimise
4) post pneumonectomy may not need chest tube

Following diagnosis, management of a chylothorax consists initially of tube thoracostomy drainage (chest tube insertion) with complete lung re-expansion and supportive measures, such as a low-fat or fat-free diet supplemented by medium-chain triglycerides and aggressive fluid, electrolyte, and nutritional replacement or correction. Often, these measures are enough to promote closure of the thoracic duct pleural fistula. If the chylothorax is caused by malignancy, primary treatment of the neoplasm may be necessary. Radiation therapy to the mediastinum has been useful in managing chylothorax secondary to lymphoma.
Conservative measures for the treatment of chylothorax generally are maintained for 1 to 2 weeks. If the chylous effusion has not responded to this management, surgical intervention is indicated. The most common procedures are ligation of the thoracic duct or mass ligation of tissue at the diaphragmatic hiatus (generally through a right thoracotomy) or direct closure of the duct injury.
- Sabiston

How well did you know this?
1
Not at all
2
3
4
5
Perfectly
47
Q

The best location for a closed thoracostomy is:

a) 2nd intercostals space (ICS), mid-clavicular line
b) 2-3rd ICS, medial to scapula
c) 5th ICS, posterior midaxillary line
d) 3-4th anterior midaxillary line

A

none of those

5th ICS, anterior to mid-axillary line
- ATLS Guidelines

The preferred approach is through the fourth, fifth, or sixth intercostal space in the mid to anterior axillary line.
- Townsend: Sabiston Textbook of Surgery, 16th ed.,

How well did you know this?
1
Not at all
2
3
4
5
Perfectly
48
Q

Lower limb embolus, 8 hours ischemia time before thrombectomy performed, which of the following will not be seen?

a) Hyperkalemia.
b) Hypercalcemia.
c) Lactic acidosis.
d) Compartment syndrome.

A

A? but you get hypocalcemia in rhabdo. i dunno

postrevascularization syndrome, regardless the cause of occlusion (ischemia, trauma, iatrogenic) or the methods used to achieve reperfusion (fibrinolysis, surgery, resuscitative therapy). This “revascularization” syndrome includes several complications, both local (explosive swelling of the limb, compartment syndrome and skeletal muscle infarction (rhabdomyolysis) and general (acidosis, hypercalcemia, hypovolaemic shock, renal, hepatointestinal and pulmonary failures, arrhythmias and cardiac arrest (multiple organ dysfunction).
http://www.ncbi.nlm.nih.gov/pubmed/9779243

49
Q

A patient with atrial fibrillation undergoes bilateral femoral embolectomy develops spike in Creatinine post op day 3 which of the following would you expect/do?

a. stop heparin
b. rbc on urinalysis
c. iv infusion of sodium bicarb

A

reperfusion rhabdomyolysis.
Better answer would be high-volume IV fluid to diurese

Rhabdomyolysis should be suspected in patients with burns, trauma, seizures, alcohol or drug intoxication, prolonged ischemia in muscle groups, and extended coma. The diagnosis can be made by the finding of elevated creatine phosphokinase and a urine microscopy study that shows prominent heme pigment without red blood cells in the urine sediment.

Rhabdomyolysis: 
ACUTE GENERAL Rx
 - Early, aggressive, high-volume IV fluid replacement with mannitol to induce diuresis to prevent acute renal failure
 - Treatment of electrolyte imbalances
 - Alkalinization of urine is controversial but appears helpful in research models
LABORATORY TESTS
 - Screening for myoglobinuria with a simple urine dipstick test using orthotoluidine or benzidine
 - Blood urea nitrogen, creatinine
 - Increased CK ( Fig. 1-247 )
 - Hyperkalemia
 - Hypocalcemia
 - Hyperphosphatemia
 - Increased urinary myoglobin
 - Pigmented granular casts
 - Hyperuricemia
Ferri: Ferri's Clinical Advisor 2010, 1st ed
50
Q

The most common reason for femoral-popliteal bypass failure at two years is:

a. Intimal proliferation
b. Progression of atherosclerosis
c. Extrinsic graft compression
d. Technical error

A

Answer: Progression of atherosclerosis

Early failure of vein grafts (within 30 days) generally represents a judgmental or technical error within the conduct of surgery. These include simple technical errors, such as a kink or twist within the graft, or failure to completely lyse the valves. Judgmental errors include the use of a small or poor-quality vein conduit or the construction of an anastomosis to an inadequate outflow artery. Intermediate failures (30 days-2 years) are generally caused by intimal hyperplastic lesions that form at anastomotic sites or valve sites within the graft. Late graft failures (beyond 2 years) are most often caused by progression of atherosclerotic occlusive disease within the inflow or outflow vessels.
- Townsend: Sabiston Textbook of Surgery, 18th ed.

51
Q

A patient with symptomatic (85%) carotid stenosis is found to have 50% stenosis of the contralateral carotid artery that is asymptomatic. Appropriate initial treatment includes which of the following?

a. Simultaneous bilateral carotid endarterectomy.
b. Staged bilateral carotid endarterectomy with 1 week interval between stages.
c. Carotid endarterectomy on the symptomatic side only.
d. Carotid endarterectomy on the side with the greatest stenosis, regardless of symptoms.
e. Optimize medical management

A

c. Carotid endarterectomy on the symptomatic side only.

I think there is newer evidence but I didn’t go looking for it

See article.
Aymptomatic >60%, Patients should be less than 75 years old with a life expectancy of more than 5 years, and an acceptable risk of surgical complications
Symptomatic >50%
Not done with 100% b/c it will cause stroke.

Generally accepted criteria based on these studies are (1) for symptomatic carotid arteries, more than 50% diameter reduction; and (2) for asymptomatic carotid arteries, more than 80% diameter reduction. Although ACAS recommended surgery for patients with more than 60% stenosis, many clinicians believe the benefit of surgery or angioplasty is not achieved for asymptomatic stenoses less than 80% because of the lower risk for stroke with lesser degrees of carotid stenosis and the relative risk of these interventions.
- Townsend: Sabiston Textbook of Surgery, 18th ed.
Amaurosis fugax is a painless, monocular loss of vision

The benefit of carotid endarterectomy in symptomatic patients with stenosis of 70 percent or greater has been established in three major studies.

52
Q

Stem describes a post carotid endart with stridor, what would you do next?
Expanding hematoma – open neck

A

(open neck)

53
Q

In a patient presenting with intermittent claudication, which of the following statements is most correct?

a. Leg pulses may be present on examination
b. Angiography is always indicated
c. Aspirin therapy may be beneficial
d. Chelation therapy may be beneficial

A

a. Leg pulses may be present on examination?

This could be neurogenic claudication.. Probably easier to answer with the real stem. And even if it was vascular claudication it could be profunda femoris claudication where you could still technically have pulses distally

Treatment

  • Physical training
  • Stopping smoking
  • Pentoxifylline
  • Nafronyl
  • Aspirin
  • Antithrombotics

Noninvasive testing may aid in predicting the location and severity of atherosclerotic occlusive disease. Noninvasive evaluation is also helpful to establish a preoperative baseline assessment for subsequent comparison postoperatively. Routine segmental Doppler pressure ABI
determinations are the standard studies, but results may be spuriously elevated in diabetic patients and those with renal failure.
[Seems to argue against B]
- Townsend: Sabiston Textbook of Surgery, 18th ed.

Patients should take aspirin 81-325 mg/day, unless there is a contraindication to aspirin therapy such as active gastrointestinal bleeding or a history of allergy to aspirin.
- Rakel: Textbook of Family Medicine, 7th ed.

Muscle tenderness, an abnormal neuromuscular examination finding, elevated skeletal muscle enzyme levels, and a normal pulse examination finding should distinguish myositis from PAD.
- Libby: Braunwald’s Heart Disease: A Textbook of Cardiovascular Medicine, 8th ed.

54
Q

Patient with ESRD who had Rt arteriovenous fistula (AVF) few 4 weeks ago. He came to ER
with sever pain and swelling of the arm. What should you do :
a. Ligate the fistula
b. Reverse the fistula
c. Antigcoagulation
d. Thromboemolectomy

A

Answer: Thromboemolectomy

Among patients with stenosis of hemodialysis access grafts, we recommend percutaneous angioplasty rather than surgery as the initial procedure (Grade 1B). Lesions unsuitable for percutaneous transluminal angioplasty can be referred for surgical revision.

Once thromboses of an AV graft have developed, the therapeutic options include percutaneous or surgical thrombectomy, thrombolytic agents, and mechanical dissolution. If these modalities are successful, a fistulogram can then be performed and detected stenoses treated with angioplasty or surgical revision. There is no optimal approach to the treatment of a thrombosed graft. The approach varies largely based upon local practice and expertise.

Whereas radiocephalic fistulas are preferred when feasible, several reports suggest that when an aggressive approach is undertaken, a high per centage of radiocephalic fistulas either fail to mature or develop thrombosis.
- Brenner: Brenner and Rector’s The Kidney, 8th ed.

55
Q

Incidental finding of asymptomatic 5.5cm aortic aneurysm, what is the risk of 5 year risk of rupture:

a. 5%
b. 10%
c. 25%
d. 50%

A
  • c. 25%
    In a large autopsy study, Darling found a 25 percent rupture rate of aneurysms between 4 and 7 cm and a 9.5 percent rupture rate for aneurysms smaller than 4 cm.
  • Schwartz, Chap 20, P. 941.
56
Q

The following statements regarding AAA’s are true except.

a. They are most commonly inferior to the renal artery.
b. At a size of 4 cm, they have a 75% chance of rupture.
c. They may present with atheroembolic disease of the leg.
d. They may present with GI hemorrhage.
e. They may present with hydronephrosis.

A

b. At a size of 4 cm, they have a 75% chance of rupture.

57
Q

Regarding AAA surgery all except?????

  • May present with abdominal pain
  • Thoracotomy may be required for proximal control
  • May present with aortocaval fistula
  • USS or CT is necessery pre-op
A

-USS or CT is necessery pre-op

58
Q

Abdominal aortic aneurysms are all of the following except:

a. most commonly infrarenal
b. more common in males than females
c. usually mycotic
d. may present to embolic findings
e. may present with back pain

A

c. usually mycotic

Answer: c. usually mycotic

The most common etiology of aneurysms is atherosclerotic degeneration of the arterial wall. The pathogenesis is a multifactorial process involving genetic predisposition, aging, atherosclerosis, inflammation, and localized proteolytic enzyme activation. Most aneurysms occur in elderly people and the prevalence of aneurysms increases with increasing age. Aneurysms can also occur in younger, genetically susceptible individuals with Ehlers-Danlos and Marfan syndromes. Other etiologies include localized infection that results in mycotic aneurysms and the rare tertiary stage of syphilis.
The most common location of aortic aneurysms is in the abdominal aorta and up to 40% of patients with infrarenal aneurysms have an aneurysm elsewhere in the aorta.
The average age of patients with abdominal aortic aneurysms is 75 years, about 10 years older than the average age of patients with clinically significant arterial occlusive disease. Abdominal aortic aneurysms are more common in men than in women with a male-to-female ratio of approximately 8:1. White males have a higher prevalence than black males or females. In 50- to 75-year-old males, aneurysms over 4 cm occur at a rate of 1.4%; in patients older than 60 years of age, the prevalence of aneurysms greater than 3 cm is 3%. Smoking is the most important risk factor and is associated with 78% of aneurysms discovered on screening. [16] Prevalence of aneurysms is approximately 10% in men with hypertension or with clinical evidence of peripheral, carotid, or coronary arterial disease. There is a definite familial incidence, with a rate of 10 to 20% and an 11.6-fold increase in relative risk in first-degree relatives of patients with abdominal aortic aneurysm. Familial aneurysms affect patients at a younger age and more women are affected.
- Sabiston

59
Q

76 year old male with colon cancer controlled locally has single met to the brain the treatment of choice is :

a. Surgery
b. Chemotherapy and radiotherapy
c. Radiotherapy and shunt
d. All of the above
e. None of the above

A

Answer: Surgery and radiotherapy

In almost every instance in which a brain tumor is suspected on the basis of the combined results of history, physical findings, and imaging studies, the first consideration is its surgical resectability. Although small meningiomas or acoustic neuromas usually do not require treatment to reduce intracranial pressure, in the majority of brain tumor patients it is appropriate to start administration of dexamethasone promptly. The purpose is to reduce intracranial pressure, which accompanies the majority of brain tumors, and to relieve neurologic symptoms caused by peritumoral brain edema
Although complete excision of a brain tumor is the ultimate goal in every case, this is not always possible. How much surgical success can be achieved with these tumors depends on several factors, including the tumor’s proximity to indispensable areas, the skill and experience of the neurosurgeon and the preoperative level of neurologic function.
It has already been mentioned that solitary brain metastases are best managed by surgical resection before radiation therapy.
Chemotherapy for brain tumors has had a disappointing record. The reasons are many, but inadequacy of drug delivery, tumor cell heterogeneity, and inherent resistance are among the important ones. Almost all efforts have been directed toward the primary brain tumors, especially the gliomas. Established brain metastases, however, respond about as well as systemic metastases do in many cancers, especially breast and small cell lung cancer.
- Goldman: Cecil Textbook of Medicine, 21st ed

In general, a symptomatic, solitary lesion that is surgically accessible should be removed. Surgery, however, should not be undertaken for multiple lesions or in patients who are severely afflicted by their primary disease. Treatment should also include preoperative dexamethasone, as in any brain or spinal cord tumor, to reduce adjacent brain edema. Whole brain irradiation is almost always indicated.
- Schwartz, Chap 26 p.1347

60
Q

A young lady with pelvic pain, mass close to the pelvis the diagnosis:
a-chordoma
b-soft tissue sarcoma
c-rectal cancer

A

Chordoma
Clival chordomas are benign tumors; but because of their critical location, local invasion, recurrence, and occasional metastatic spread, their prognosis is similar to that of malignant tumors.
For sacrococcygeal chordomas, the average survival rate from the onset of symptoms is usually 5.7 years. Metastasis is rare, but when it occurs, it is most often to the lymph nodes, lungs, and liver. In 5-40% of patients with spinal and sacrococcygeal chordoma, metastatic lesions have been reported.3 Recurrence after curative resection is frequent and may result in a slow but relentless and fatal progression of the disease as a result of invasion of local pelvic structures. This is one of the most difficult tumors for the surgeon to treat. Infection is a frequent complication.
- http://emedicine.medscape.com/article/339169-overview

Incidence 1 in 1 000 000 …

61
Q

Patient is brain dead with astrocytoma, he is otherwise healthy. which organs could be donated: :

a. Donate all organs with out restriction
b. Donate the kidneys only
c. Donate the liver only
d. Donate the heart only
e. Not acceptable donor

A

a. Donate all organs with out restriction

The pattern of spread of primary CNS neoplasms differs from that of other tumors: even the most highly malignant gliomas rarely metastasize outside of the CNS. The subarachnoid space provides a pathway for spread, so that seeding along the brain and spinal cord can occur in highly anaplastic as well as in well-differentiated neoplasms that extend into the CSF pathways.
- Cotran: Robbins Pathologic Basis of Disease, 6th ed., Copyright © 1999 W. B. Saunders Company

Patients with primary central nervous system (CNS) tumor have been accepted for organ donation because these tumors very rarely spread outside the CNS. However several case reports of CNS tumor transferral with organ transplantation recently challenged this attitude. Some risk factors for extraneural spread of CNS tumors have been determined, but the absence of risk factors does not exclude the possibility of metastases. To our knowledge, 13 cases of CNS tumor transferral with organ transplantation (one heart, three livers, eight kidneys, one kidney/pancreas) have been reported in the literature. Even if no prospective evaluation of the CNS tumor transmission risk with transplantation has been undergone, this risk may be estimated between a little more than 0% and 3% from retrospective series. The authors consider that patients with CNS tumor should be accepted as donors as long as the risk of dying on the waiting lists is significantly higher than the tumor transferral risk. Therefore the authors would have no restriction for transplanting organs from donors with benign or low-grade CNS tumor. For high-grade tumors, the authors would consider these donors as “marginal donors,” and balance the risk of tumor transmission with the medical condition of the recipient.
Organ donors with primary central nervous system tumor.
- Detry O - Transplantation - 15-Jul-2000; 70(1): 244-8

62
Q

Which of the following paediatric tumours arises from the sympathetic nervous system :

a. Wilm’s tumour
b. Neuroblastoma
c. Medulary blastoma
d. Teratoma
e. None of the above

A

Neuroblastoma

63
Q

During a cervical spine operation the vertebral artery was transected but ligated. What is the MOST common outcome?

a. No neurological sequalae
b. Blindness
c. Hemispheric stroke
d. Weakness of the tongue
e. Hearing loss and vertigo

A

a. No neurological sequalae

In fact, several studies have demonstrated vertebral artery occlusion in approximately 20 percent of nonpenetrating cervical trauma. [0540] [1330] [1370] Fortunately, most of these patients are not symptomatic, which is likely due to a usually generous intracranial and extracranial collateral circulation.
In the rare case in which ligation is required for hemostasis, intraoperative angiography may be performed to assess the ability of the patient to tolerate such a procedure. However, in these cases, such studies are probably impractical, and, furthermore, some evidence indicates that sacrifice of one vertebral artery does not lead to permanent neurologic sequelae in most patients.
- Browner: Skeletal Trauma, 4th ed.

3% stroke rate (for transected and ligated)

64
Q

What is the brain tumor with the best prognosis?

a) meningioma
b) astrocytoma
c) ependymoma

A

a) meningioma

You can shuck out a meningioma like a golf ball.

often benign

65
Q

What is the most common benign brain mass?

A

Answer: meningioma

Neuroepithelial tumours account for 50–60 per cent of all primary brain tumours and represent a broad spectrum of neoplasms arising from or sharing morphological properties of neuroepithelial cells. They include glial neoplasms; choroid plexus tumours; tumours with predominant neuronal phenotype (ganglioglioma, dysembryoplastic neuroepithelial tumour and neurocytoma); pineal tumours and embryonal tumours (neuroectodermal tumours, medulloblastoma).
Astrocytomas account for approximately 75 per cent of glial tumours.
Overall, meningiomas represent the commonest nonglial intracranial neoplasm, accounting for approximately 20 per cent of all primary intracranial tumours.
- Adam: Grainger & Allison’s Diagnostic Radiology, 5th ed.

66
Q

A 45 year old female presents with a 2cm lump on the left anterior surface of her neck. She was treated with radiation for acne when she was an adolescent. After completing a physical exam and ordering the appropriate bloodwork, what is the next appropriate investigation?

a) Ultrasound
b) CT scan
c) Fine needle aspirate
d) Core needle biopsy

A

c) Fine needle aspirate

FNA biopsy has become standard in making both diagnostic and management decisions on neck masses. When the FNA biopsy specimen tests positive for carcinoma and the clinical examination and endoscopy do not reveal a primary tumor site, open excisional biopsy is the next step in confirming or diagnosing the mass.
- Cummings: Otolaryngology: Head & Neck Surgery, 4th ed.

Needle biopsy with cytologic examination has become the prime diagnostic tool for neck masses. Multiple aspirations with a small-gauge needle is the recommended technique. The exception to this interventional step is the patient who has a pulsatile mass or one who has a palpable thrill or audible bruit associated with the neck mass.
- The neck mass. McGuirt WF - Med Clin North Am - 01-Jan-1999; 83(1): 219-34

67
Q
A patient presents with a mass anterior to his ear. What is the next MOST appropriate step? 
A. FNA
B. CT scan
C.  MRI
D. Surgical excision
A

A. FNA

Fine-needle aspiration biopsy of salivary gland neoplasms is a simple and accurate aid in diagnosis. The information gained by cytologic examination can be helpful for patient counseling, preoperative consultation, and surgical planning

68
Q
The initial treatment of thyrotoxic storm may include all of the following EXCEPT:
A. Propranolol
B. Urgent thyroidectomy
C. Propylthiouracil (PTU)
D. Lithium
A

B. Urgent thyroidectomy

Although lithium doesn’t seem to be used much/recommended, the Keyword that makes urgent thyroidectomy the answer is “urgent” - you want to treat the thyrotoxicosis first before going for surgery..

For patients with clinical features of thyroid storm, we begin immediate treatment with a beta blocker (propranolol in a dose to achieve adequate control of heart rate, typically 60 to 80 mg orally every four to six hours), a thionamide, glucocorticoids (hydrocortisone, 100 mg intravenously every eight hours), and cholestyramine (4 g orally four times daily). One hour after a thionamide is given, we administer iodine (SSKI, five drops [20 drops/mL, 38 mg iodide/drop] orally every six hours, or Lugol’s solution, 10 drops [20 drops/mL, 8 mg iodine/drop] every eight hours)

For patients with life-threatening thyroid storm admitted to an ICU, we suggest propylthiouracil (PTU) (200 mg orally every four hours) rather than methimazole as initial therapy (Grade 2B). PTU blocks T4 to T3 conversion and results in lower serum T3 levels for the first several days of treatment. However, for severe but not life-threatening hyperthyroidism, methimazole (20 mg every six hours) may be preferred because of its longer half life, lower risk of hepatic toxicity, and because it ultimately restores euthyroidism more quickly than PTU. Patients initially treated with PTU should be transitioned to methimazole before discharge from the hospital.

For patients with contraindications to thionamides who require urgent correction of hyperthyroidism, surgery is the treatment of choice. Patients who are to undergo surgery require preoperative treatment of thyrotoxicosis. We typically treat with beta blockers (if not contraindicated, propranolol 60 to 80 mg every four to six hours), glucocorticoids to inhibit conversion of T4 to T3 (eg, dexamethasone, 1 to 2 mg every six hours), bile acid sequestrants (eg, cholestyramine 4 g orally four times daily), and, in patients with Graves’ disease, iodine (SSKI, five drops [50 mg iodide/drop] orally every six hours, or Lugol’s solution, 10 drops [8 mg iodide/iodine per drop] every eight hours).

Lithium has also been given to acutely block the release of thyroid hormone. However, its renal and neurologic toxicity limit its utility.

69
Q

The most common cause of death following a tracheostomy is:

a. bleeding
b. airway loss due to accidental decannulation or tube obstruction
c. pneumothorax
d. tracheo-esophageal fistula

A

airway loss due to accidental decannulation or tube obstruction

Most life threatening cause (which is rare) tracheo-inomminant fistula.

70
Q

35year old female in the third trimester found to have a thyroid mass, the biopsy shows papillary cancer. The most appropriate management is:

a. Wait until delivery and do thyroidectomy
b. Total thyroidectomy now
c. Terminate pregnancy and do thyroidectomy
d. Chemotherapy
e. Radiotherapy

A

Answer: Wait until delivery and do thyroidectomy

After an appropriate diagnosis, surgical resection remains the primary mode of treatment. The timing of this intervention remains an important decision. A retrospective review suggests equivalent outcomes in patients who undergo thyroidectomy during pregnancy and those who wait until the postpartum state. Radionuclide thyroid ablation is contraindicated during pregnancy.
- Goldman: Cecil Medicine, 23rd ed.

From Motherrisk
Surgery remains treatment of choice for well differentiated thyroid (papillary and follicular) cancer. Due to the relatively indolent course but potential impact on long term survival, therapy during pregnancy can be tailored to the stage of pregnancy. For those presenting in the first trimester, surgery and general anaesthesia can be delayed until the second trimester to reduce the risks associated with surgery to the fetus. A few reports suggested that the risk of fetal loss related to surgery is minimal37.For those patients who present in the later stages of pregnancy with a well-differentiated tumor, surgery can usually be delayed safely until the post partum period

71
Q

During neck dissection, IJ was entered. What is the most important initial intervention:

a. Ligate the vain
b. Primary repair
c. Put the patient in trendelenberg position
d. Decrease vent. Pressure

A

c. Put the patient in trendelenberg position

Local pressure is probably a better answer

The most commonly injured vascular structure is the internal jugular vein, which is amenable to simple lateral repair or ligation. (Sabiston)
If evidence of air embolism then go to trendelenburg and left lateral decubis.

Air embolus
This is a rare event which can occur following injury to the IJV. Large emboli can produce sudden falls in end-tidal carbon dioxide and arterial blood pressure. A pre-cordial Doppler probe may detect the characteristic murmur of venous embolus. Local pressure should be applied and the anaesthetist informed so the patient can be placed in the Trendelenburg position and rotated to the left. In severe cases attempts can be made to pass a catheter and aspirate air from the right side of the heart. Hyperbaric oxygen therapy, were available, is the ultimate and effective treatment.
- Kerawala et al. Prevention of complications in neck dissection. Head & Neck Oncology 2009, 1:35

72
Q

A 65 patient with untreated hypothyroidism going to surgery. She is at risk for:

a. Pulmonary complications.
b. No increase risk.
c. Cardiac complications
d. Bleeding complications

A

c. Cardiac complications

Severe symptomatic hypothyroidism has been associated with several perioperative complications, including intraoperative hypotension, congestive heart failure, cardiac arrest, and death. Elective surgery should be delayed in patients with severe hypothyroidism until adequate thyroid hormone replacement can be achieved. If emergency surgery is required in such patients, intravenous T3 or T4 and glucocorticoids should be administered perioperatively. Conversely, patients with asymptomatic or mild hypothyroidism generally tolerate surgery well, with only a slight increase in the incidence of intraoperative hypotension; surgery need not be delayed for the month or more required to ensure adequate thyroid hormone replacement.

73
Q

In regards to the facial nerve, while completing excisions of lesions on the face and neck one must avoid the facial nerve

a) as it exits the stylomastoid foramen
b) as it runs suprafascially on the parotid gland
c) as it crossed over the infraorbital rim
d) as it passes 2 finger widths below the angle of the jaw, deep to the platysma
e) as it enters the buccinator

A

Facial Nerve Anatomy
The facial nerve crosses lateral to the styloid process and penetrates the parotid gland. The nerve lies in a fibrous plane that separates the deep and superficial lobes of the parotid gland.
In the lower face, the facial nerve always runs deep to the platysma and SMAS and innervates the muscles on their undersurfaces (except for the buccinator, levator anguli oris, and mentalis muscles)
The mandibular (or marginal) division lies along the body of the mandible (80%) or within 1-2 cm below (20%). This is a critical landmark in head and neck surgery. The marginal branch lies deep to the platysma throughout much of its course. It becomes more superficial approximately 2 cm lateral to the corner of the mouth and ends on the undersurface of the muscles. Injury to the marginal branch results in paralysis of the muscles that depress the corner of the mouth.
- http://emedicine.medscape.com/article/835286-overview

The superficial branches run beneath the skin and above the superficial muscles of the face, which they supply: some are distributed to the Procerus, joining at the medial angle of the orbit with the infratrochlear and nasociliary branches of the ophthalmic.
The lower deep branches supply the Buccinator and Orbicularis oris, and join with filaments of the buccinator branch of the mandibular nerve.3
- http://en.wikipedia.org/wiki/Buccal_branch_of_the_facial_nerve

74
Q

A 57 year old male undergoes a resection of his left parotid gland for adenocarcinoma. Upon waking he is unable to smile on his left side.
Which of the following nerves was most likely damaged:
a) left facial nerve - zygomatic branch
b) left facial nerve - marginal mandibular branch
c) left facial nerve - buccal branch
d) left facial nerve - cervical branch
e) left facial nerve - temporal branch
What is the answer then?

A

b) left facial nerve - marginal mandibular branch

Elevating the corner of the mouth is mostly from:
- Zygomatic + Buccal
o Zygomaticus major (Elevates corners of mouth)
- Buccal
o Zygomaticus minor (Elevates upper lip)
o Levator labii superioris (Elevates upper lip and midportion nasolabial fold)
o Levator labii superioris alaeque nasi (Elevates medial nasolabial fold and nasal ala)
o Risorius (Aids smile with lateral pull)
o Buccinator (Pulls corner of mouth backward and compresses cheek)
o Levator anguli oris (Pulls angles of mouth upward and toward midline)

The marginal mandibular innervates the mentalis, which pulls skin of chin upward, as well as depressing the corner and mid lip.
- Bailey. Head and Neck : Otolaryngology

75
Q

Which blood vessel may rupture in a person with a tracheostomy (??something like that)

a) aorta
b) innominate
c) inferior thyroid
d) jugular

A

The tracheoinnominate fistula may result from prolonged cuff erosion inferiorly and anteriorly to the trachea.

76
Q

A 4 yo male develops left parotid tenderness 24 hours after an unrelated minor surgery. Pus can be expressed from Stensen’s duct. Which of the following is the most appropriate antibiotic?

a) gentamicin
b) tetracycline
c) cefazolin
d) chloramphenicol
e) erythromycin

A

The most common pathogen for post-op parotitis is S. aureus. But neither Vanco nor Clox were options here. First generation cephalosporin is therefore most appropriate.

If the diagnosis of parotitis is being entertained, the patient receives IV, high-dose, broad-spectrum antibiotics with good coverage of Staphylococcus (the most common agent cultivated from this disease).
- Townsend: Sabiston Textbook of Surgery, 18th ed.

Suppurative parotitis, most often caused by Staphylococcus aureus or gram-negative organisms, usually occurs in the postoperative period, in premature newborns, or in debilitated patients with poor oral intake. The gland is warm, hard, and extremely tender; the overlying skin is erythematous. Massage of the parotid expresses purulent drainage from Stensen’s duct.
- Mandell: Mandell, Douglas, and Bennett’s Principles and Practice of Infectious Diseases, 7th ed.

77
Q

All of the following are complications of total thyroidectomy, except:

a) hoarseness
b) seroma
c) hematoma
d) hypocalcemia
e) airway obstruction

A

Answer: seroma

Unilateral lobectomy virtually never causes permanent hypocalcemia but can cause vocal cord paralysis in as many as 3% of patients. Near-total thyroidectomy causes temporary hypocalcemia in 7 to 10% of patients and permanent hypocalcemia in 0.5 to 1%; temporary vocal cord paralysis occurs in about 1 to 2%.
A total extracapsular thyroidectomy may lead to hypoparathyroidism in as many as 30% of individuals, an unacceptable complication rate for patients with indolent malignancy. In addition, vocal cord paralysis is more common after such a procedure. The experience of the surgeon is important in terms of the finer technical points of thyroidectomy, including preservation of the external branch of the recurrent laryngeal nerve, which is important in the fine regulation of voice pitch. This is especially pertinent for individuals who depend on the voice for a livelihood.

78
Q

The most common cause of Carpal tunnel syndrome is:

a) tenosynovitis
b) trauma
c) Rheumatoid arthritis
d) Amyloidosis

A

This is a dumb question.

The pathophysiology of CTS is multifactorial. Increased pressure in the intracarpal canal plays a key role in the development of clinical CTS [4].

While the precise etiology of increased carpal tunnel pressure in CTS is uncertain, experimental evidence suggests that anatomic compression and/or inflammation are possible mechanisms. Increased pressure in the carpal tunnel can injure the nerve directly, impair axonal transport, or compress vessels in the perineurium and cause median nerve ischemia

79
Q

All the following are risk factors for carpal tunnel syndrome EXCEPT:

a. DM
b. COPD.
c. Hypothyroidism.
d. Acromegaly

A

COPD

Uptodate
Obesity
Female gender
Pregnancy
Diabetes
Rheumatoid arthritis
Hypothyroidism
Connective tissue diseases
Preexisting median mononeuropathy
Genetic predisposition
Aromatase inhibitor use
Workplace factors
80
Q

A patient has volar wrist trauma with several structures injured including an artery and flexor tendon. What do you do next?

a) Saline wash, debride and close, then proceed to the OR
b) Clean up,and debride, leave open and splint and ask the patient to return to see the surgeon the next day

A

a) Saline wash, debride and close, then proceed to the OR

81
Q

A 40 y old assembly line worker presents to the ER with a mangled right distal forearm. He describes that the paramedics had a very difficult time controlling the bleeding but finally managed with the tightly bound dressing the patient has around the distal forearm. The most appropriate next step is:

a) send the patient for X-rays
b) remove the dressing and inspect the wound
c) perform a physical examination of the hand
d) proceed immediately to the OR to remove the dressing and examine the patient under anesthesia

A

???

c) physical exam

followed by

d) proceed immediately to the OR to remove the dressing and examine the patient under anesthesia

82
Q

What structure is at risk during a fasciotomy of the anterior compartment of the lower leg?

a. Superficial peroneal nerve
b. Deep peroneal nerve
c. Posterior tibial nerve
d. Posterior tibial artery

A

a. Superficial peroneal nerve

The surgeon then identifies the crural fascia between the anterior and lateral compartments and divides it along the entire length of the incision, taking care to avoid injury to the lateral peroneal nerve in the superior aspect of the incision, the most common iatrogenic injury during an anterior fasciotomy.
- Townsend: Sabiston Textbook of Surgery, 18th ed.

Deep peroneal nerve courses anteriorly around fibula, taking a sharp turn as it rounds the fibular neck, to enter anterior compartment of leg.
Superficial peroneal nerve supplies lateral compartment of leg, first passing between peroneus longus and passes in a straight line from the common peroneal nerve
http://www.wheelessonline.com/ortho/peroneal_nerve

Uptodate
Neurovascular injury — Injuries to arteries and nerves due to technical issues are specific to the fasciotomy site. In one study, injury to the superficial peroneal nerve occurred in 6 percent of patients undergoing emergent leg fasciotomy for trauma [42]. This injury is generally due to failure to recognize the nerve while extending either the anterior or lateral compartment fascial incision proximally. Superficial peroneal nerve injury is avoided first by knowledge of the normal and variant anatomy (figure 3) of this nerve and stopping the proximal fascial incision of the anterior and lateral compartment 4 to 5 cm distal to the fibular head. (See ‘Superficial peroneal nerve’ above.)

The peroneal artery is at risk for injury with the single-incision technique for lower leg fasciotomy, particularly if a fibulectomy is performed. (See ‘Single incision fasciotomy’ above.)

83
Q

All are signs of suppurative tenosynovitis except:

a. Takes flexion position
b. Redness of the finger
c. Tender when you palpate the flexor site
d. Tenderness in passive extension of the fingers

A

b. Redness of the finger

Kanavel’s signs: know them

An infection within the flexor tendon sheath may be the result of the spread of adjacent pulp infections or puncture wounds in the flexor creases. Although the flexor sheath usually is involved, the radial and ulnar bursae may be involved as well. Kanavel considered tenderness over the involved sheath, rigid positioning of the finger in flexion, pain on attempts to hyperextend the fingers, and swelling of the involved part to be the four cardinal signs of suppurative tenosynovitis. Of these, tenderness over the flexor sheath is considered the most significant.
- Canale & Beaty: Campbell’s Operative Orthopaedics, 11th ed

84
Q

Which pathogen is most commonly associated with suppurative tenosynovitis?

a) S. Aureus
b) S. pneumoniae
c) S. Epidermis

A

a) S. Aureus

Infectious and Inflammatory Flexor Tenosynovitis
The primary inciting event of infectious flexor tenosynovitis (FT) usually is penetrating trauma. Most infections are caused by native skin flora, including both Staphylococcus and Streptococcus species. The most common organism that causes infectious FT is Staphylococcus aureus.
- http://emedicine.medscape.com/article/1239040-overview

85
Q

The most common site for ganglia is:

a. Radial aspect of the dorsum of the wrist
b. Flexor aspect of the wrist
c. Forearm
d. Dorsum of the foot

A

a. Radial aspect of the dorsum of the wrist
a. Radial aspect of the dorsum of the wrist

Ganglion cysts can occur at any joint or tendon sheath, but they most often present in the dorsum of the wrist at the scapholunate joint, followed by the volar wrist. They can also occur in the flexor tendon sheaths, and when located at the DIP joints, they are termed mucous cysts2,8 Of the dorsal wrist ganglia, 75% connect with the dorsal scapholunate interosseous ligament.2 Many patients who have had asymptomatic cysts for months or even years can present with newly developed pain or limitation of activity.
- http://emedicine.medscape.com/article/1243454-overview

Plastic Seminar
O Location :
• Dorsal wrist (60-70%); Commonly over scapholunate ligament
•Volar wrist (18-20%); Between FCR and APL at scaphotrapezoid junction

86
Q

Langer’s lines except:

a. Perpendicular to muscle
b. Created by muscle pull and joint movement
c. Genetically predetermined
d. Incision cross is associated with hypertrophic scar

A

I think this question is misremembered.

Langer’s are parallel to muscle
I don’t think it’s genetically predetermined (determined by muscle mvt)

They correspond to the natural orientation of collagen fibers in the dermis, and are generally parallel to the orientation of the underlying muscle fibers.

Widened scar formation is thought to result from wound edge separation with tension perpendicular to the healing skin wound. A state of tension exists naturally in skin; wounded skin gapes and becomes elliptical rather than round. Although Dupuytren first noted this property of skin, Langer received most of the credit. Langer studied the direction of these ellipses by stabbing a round-tipped awl into hundreds of cadavers. When a wound is closed opposite to the lines of tension, the chance of widened scar formation is increased.
- http://emedicine.medscape.com/article/1298541-overview

Langer’s lines are lines which can be extrapolated by connecting linear openings made when a round pin is driven into the skin of a cadaver, resulting from the principal axis of orientation of the subcutaneous connective tissue (collagen) fibres of the dermis; they vary in direction with the region of the body surface. Skin tends to be much stiffer along these lines rather than across them.
These lines were developed by Karl Langer, an anatomy professor, from cadavers in rigor mortis. Kraissl, however, preferred lines oriented perpendicular to the action of the underlying muscles. Later, Borges described relaxed skin tension lines, which follow furrows formed when the skin is relaxed and are produced by pinching the skin.
Langer’s lines are normal, permanent skin creases. They are clinically significant as incisions made parallel to them heal well and produce less visible scarring. N.B These lines are in the dermis but they are reflected in the skin surface in some sites like : Face , Base of the neck

87
Q

Patient requires fasciotomy; appropriate closure includes all, EXCEPT:

a) Primary closure in 3-7 days
b) Closure with STSG in 3-7 days
c) Immediate closure of overlying skin
d) Healing by secondary intention

A

?immedite closure of skin (skin can cause compartment syndrome if really tight)

Removal of necrotic debris is the first step in reducing the bacterial content of the wound. Bacterial counts of greater than 100,000 (105 ) colonies per gram of tissue in pressure ulcers correlate well with impaired wound healing and wound graft failure. Wounds with eschar or large amounts of necrotic debris are best debrided surgically.

Fasciotomy is the preferred operative therapy for compartment syndrome involving forearm or leg. Anterior and lateral compartments should be decompressed through an anterior lateral incision, and posterior compartments through a posteromedial incision.
These longitudinal incisions are usually left open for delayed primary closure or skin grafting.
- Greenfield

88
Q

Paraplegic patient times 15 years, lives with wife and children and owns a bookstore, has pressure ulcer with exposed ischial tuberosity and extensive undermining. Best treatment
A – saline wet-to-dry dressings
B – surgical debridement and dressings
C – enzymatic debridement and colloid dressing
D – autolytic debridement and sponge dressing
E – surgical debridement and myocutaneous flap coverage

A

E – surgical debridement and myocutaneous flap coverage

89
Q

Silicone implants cause

a) fibrosis
b) tissue capsule
c) silicon granulomas
d) connective tissue disease

A

b) tissue capsule

Complications associated with the use of breast implants can occur in the immediate perioperative period or years later. [3] These complications include exposure, extrusion, or infection of the implants. [1] [57] Careful attention when closing the submuscular pocket and skin can help avoid these problems. Longer-term problems also include asymmetry, capsular contracture, malposition of the implant, rupture, and pain.

Capsules of tightly-woven collagen fibers form as an immune response around a foreign body (e.g. breast implants, pacemakers, orthopedic joint prosthetics), tending to wall it off. Capsular contracture occurs when the capsule tightens and squeezes the implant. This contracture is a complication that can be very painful and distort the appearance of the implanted breast.
Fibrosis of breast tissue after XRT is common and an increase in capsular contracture rates would be expected.
- http://en.wikipedia.org/wiki/Breast_implant

90
Q
How long can amputated digits survive prior to microvascular reimplantation if properly stored on ice?
A. 6h 
B. 8h
C.10h
D. 12h
A

Answer: 12h (debatable)

Because irreversible necrotic changes begin in muscle after 6 hours of ischemia without cooling (at 20°C to 25°C), it is preferable to begin the replantation of parts amputated proximal to the palm within this time. With cooling (to 4°C), this time may be extended to 12 hours. For parts with no muscle (digits), the allowable warm ischemia time may be 8 hours or more. With cooling, this has been extended to longer than 30 hours.
- Canale & Beaty: Campbell’s Operative Orthopaedics, 11th ed.

Ischemia time is also an important consideration when evaluating a patient for replantation. For amputated digits, more than 12 hours of warm ischemia is a relative contraindication. Promptly cooling the part to 4°C dramatically alters the ischemia factor, but even ischemia exceeding 24 hours does not necessarily preclude successful digital replantation. Ischemia time is more crucial for replantation above the proximal forearm, and reimplantation is not considered after more than 6 to 10 hours of warm ischemia time.
- Townsend: Sabiston Textbook of Surgery, 18th ed.

Classical textbook teaching states 6hours warm; 12 with cold

Plastic Seminar
Contra-indications to replant.

Ischemia time

  • proximal to carpus: > 6hrs warm; >12 hrs cold
  • digits: > 12 hrs warm; > 24-30 hrs cold
91
Q

Which an absolute indication for reimplantation
a- thumb
b- ringfinger
c- 5th finger
d- metacarpal carpal joint avulsion of 3rd digit

A

a- thumb

Indications for replantation of amputated parts are as follows:

  1. Whenever possible for a thumb amputation (it provides >40% of the overall hand function)
  2. Single digits that have been amputated distal to the FDS insertion (a manual worker may likely desire revision of amputation and desires to return to work quickly)
  3. Multiple digits
  4. Most amputations in children, including single digit amputations
  5. Guillotine-sharp clean amputations at the hand, wrist, or distal forearm
    - Townsend: Sabiston Textbook of Surgery, 18th ed.
92
Q

The first tissue to show ischemia in a flap is:

a. Skin
b. Muscle
c. Fascia
d. Fat
e. Nerve

A

b. Muscle

In addition, the acceptable ischemia time is shortest with the enteric flaps because of their high tissue oxygen and nutrient demand.
- Townsend: Sabiston Textbook of Surgery, 18th ed.

Warm ischemic time: muscle, bone, skin
Cold ischemic time: muscle, skin, bone
I think I disagree with above. I think bone is the most resistant no matter what the temperature is

93
Q

Warm ischemic tolerance is best for which of the following:

a) gut
b) muscle
c) bone
d) skin

A

Answer: bone

The time that an amputated part can survive before replantation has not been determined. After 6 hours, additional delay may decrease the success rate of revascularization and lead to diminished function. Skin, bone, tendons, and ligaments tolerate ischemia much better than do muscle and connective tissue.
- Roberts: Clinical Procedures in Emergency Medicine, 5th ed.

Skin, subcutaneous tissue, muscle, and bone have different ischemic tolerances. Skin and subcutaneous tissue is relatively resistant to anoxia and can tolerate warm ischemia for 4-6 hours and cold ischemia for up to 12 hours.[2, 3] Skeletal muscle is less tolerant to ischemia than skin. Muscle can tolerate warm ischemia for up to 2 hours and irreversible damage to the microcirculation begins at 6 hours, even when under cold ischemia.[4, 5] Bone is more resistant to anoxia and can tolerate up to 24 hours of cold ischemia.

94
Q

3 hours post flap – congested and dark when poked dark blood
what is management?
a) leeches
b) exploration

A

If a problem is suspected, the first response must include removal of enough sutures at the bedside that will relieve pressure on the flap. The standard of care dictates a rapid return to the operating room to release tension, evacuate any fluid collections, eliminate sources of vascular pedicle kinking, and examine and possibly revise the arterial or venous anastomosis. Numerous pharmacologic agents have been used to manipulate vascular tone, the clotting cascade, and to reduce the ill effects of the inflammatory mediators liberated through the arachidonic acid pathway. Leech therapy may on occasion salvage flaps that suffer from significant venous congestion.
- Townsend: Sabiston Textbook of Surgery, 18th ed

Flap failure shows a biphasic distribution

  • Failure in the first 12-72 hours is almost exclusively due to technical difficulties at the anastomosis and usually at the venous side
  • Late failures (>4d) are less common and are primarily related to infection

CONSERVATIVE MANAGEMENT SHOULD BE FIRST ATTEMPTED
▪ Reposition patient to ↓ pedicle compression
▪ Remove all dressings
▪ Check for tightness of skin closure, release sutures if necessary
▪ If edema present → elevate involved area, release sutures
▪ Assess hydration status →rule out hypovolemia
▪ Prepare patient for possible surgical re-exploration

EMERGENT SURGICAL RE-EXPLORATION IF:
▪ Conservative measures fail after 30-60 min
▪ Definite vascular compromise (artery or vein)
▪ Hematoma is diagnosed

TEMPORIZING MEASURES IF DEFINITE VASCULAR PROBLEM, BUT
PATIENT CANNOT BE IMMEDIATELY BROUGHT TO THE OPERATING ROOM
▪ Arterial thrombosis:
▫ No substitute for immediate re-exploration
▫ Soft tissue will tolerate 4 h of warm ischemia
▫ Inability to reperfuse flap (no reflow phenomenon) and tissue necrosis occurs after this
▪ Venous thrombosis:
▫ Flap elevated to above level of heart
▫ Leeches

95
Q
How does leech therapy work in a congested and edematous flap?
A. Decreases platelet aggregation
B. Alters blood flow dynamics
C. Alters coagulation cascade
D..Endothelial damage
A

Answer: Alters coagulation cascade (thrombin inhibition)

Leech-derived anticoagulants are small molecules that directly inhibit thrombin (factor IIa). Halting the coagulation cascade at this stage prevents fibrinogen cleavage into fibrin, which is necessary for cross-linkage and subsequent clot formation. Hirudin is the actual 65–amino acid anticoagulant protein found in leech salivary glands.
- Shannon: Haddad and Winchester’s Clinical Management of Poisoning and Drug Overdose, 4th ed.

Leech saliva helps reestablish blood flow to reattached body parts by means of a vasodilator, provides a numbing anesthetic (Hirudinae)
- No source … suspect

Hirudin (direct thrombin inhibitor)
• hirudin secreted on biting
• direct enzyme inhibitor of thrombin
• inhibits conversion of fibrinogen to fibrin
other substances secreted as well which act as local anesthetics and vasodilators

Hirudin, obtained from the saliva of leeches, is a single-chain polypeptide composed of 65 amino acids, with three disulfide bonds and 8000 to 9000 MW. It is highly specific for thrombin inhibition. Hirudin has a short half-life and is excreted unchanged in the urine. Hirudin has no natural inhibitors, compared with heparin, which has natural inhibitors platelet factor 4 and fibrin II–monomer. As an inhibitor of thrombin, hirudin prevents conversion of fibrinogen to fibrin; thrombin-catalyzed activation of factors V, VIII, and XIII; and importantly, thrombin-induced platelet aggregation. In addition to its small size and high potency for thrombin, hirudin has a dominant antiplatelet effect, even with platelet-rich thrombi. Hirudin has been found to be more effective than heparin in reducing platelet deposition and mural thrombus at similar aPTT levels.115 Levels of aPTT of 2 to 3 control (0.7 to 1 mg/kg hirudin) are effective in limiting arterial thrombosis and platelet deposition. Hirudin prevents thrombus growth at both high and low shear rates of blood flow and has even been found to stop thrombus growth in severe stenoses.
- Greenfield

96
Q

What is the most reliable indicator of flap perfusion?

a) Heart rate
b) Urine output
c) Skin color
d) Skin temperature

A

c) Skin color

McNeil said something abuot pinprick.. but you won’t have any sensory innervation with a free flap

Despite numerous techniques, such as Doppler probing and temperature monitoring, clinical assessment remains the gold standard for free flap monitoring. The most common cause of flap failure is venous congestion.
- Townsend: Sabiston Textbook of Surgery, 18th ed.

After successful flap design and implementation, monitor the flap for viability as early recognition of ischemia is important in preventing subsequent flap necrosis, thus flap failure. Clinical observation is the best method to assess a flap. An extremely pale flap may signify arterial insufficiency, while a blue flap may be secondary to a failure of venous outflow. Two additional tests often used to assess viability are capillary refill and warmth. Assessment of bleeding from the flap after stabbing it with a small needle is believed to be one of the most reliable methods of clinical assessment.
In addition to clinical assessment, objective tests such as pH monitoring and transcutaneous oxygen tension (PO2)17,18 can aid in early detection of flap ischemia. Doppler ultrasound is used commonly, and laser Doppler is increasingly useful1 Surface temperature monitoring is another dependable technique. Fluorescein dye and illumination with a Wood lamp is useful; however, it is cumbersome, and adverse reactions to the dye have been reported.
- http://emedicine.medscape.com/article/1284569-overview

97
Q

Why do you “mesh” partial thickness skin grafts?

A

Answer: allow exudative debris to leave
Reduces donor site morbidity
decreases potential space that could “lift it off”

98
Q

The advantage of a split-thickness skin graft to a full thickness skin graft is that:

a) it scars less and blends in with the surrounding skin
b) better over bony
c) better scar contraction

A

better take

99
Q

In comparison of full thickness graft, partial thickness graft:

a. Better take over contaminated surface
b. Less contraction
c. Less pigmentation
d. More durable
e. None of the above

A

a. Better take over contaminated surface

In general, full-thickness grafts contract less than do split-thickness grafts. The deep dermal component is able to suppress myofibroblast function by an unknown mechanism.

FTSG

  • more of the characteristics of normal skin, including color, texture, and thickness,
  • greater collagen content, dermal vascular plexus, and epithelial appendage
  • less contraction while healing.
  • limited to relatively small, uncontaminated, well-vascularized wounds and thus do not have as wide a range of applications as split-thickness grafts

STSGs
-require less ideal conditions for survival and have a much broader range of application
-resurface large wounds, line cavities, resurface mucosal deficits, close flap donor sites, and resurface muscle flaps.
-disadvantages
-more fragile, especially when placed over areas with little underlying soft tissue support, and usually do not withstand subsequent radiation therapy.
-They can contract significantly during healing.
They tend to be hypo- or hyperpigmented, particularly in darker-skinned individuals. Their thinness, abnormal pigmentation, and frequent lack of smooth texture and hair growth make STSGs more functional than cosmetic.

100
Q
In a wound, least contracture is seen with
a – FTSG
b – PTSG
c – xenograft
d – aminionic sack graft
A

a – FTSG

Depends what contracture it asks for

Primary contracture: when you cut it out of the donor site, how much the graft itself bouces and shrinks right then and there- directly proportional to elastin content, so more in FTSG
Secondary contracture : how much it contracts once healed/healing - more in STSG

FTSG = full thickness skin graft = entire epidermis and dermis
PTSG = partial thickness skin graft = epidermis and variable amount of dermis (hair follicles, sweat glands are left behind)

A skin graft that includes the entire dermis is called a full-thickness graft, whereas grafts that include only a portion of the dermis are referred to as partial-thickness grafts.. In selecting the type of skin graft to be used, one must consider wound contraction. The thicker the graft, the less wound contraction. A wound that receives no graft but is just left to heal by contraction and reepithelialization will have the greatest amount of wound contraction. Full-thickness skin grafts are usually used in the face to reduce wound contraction.
- Sabiston

Xenograft (from porcine skin) do not last long and amniotic sack graft rarely used (Swartrz).

101
Q

Which of the following statements is TRUE regarding skin grafts?

a. Secondary contraction is more likely to occur after adequate healing of a split-thickness skin graft than a full-thickness skin graft.
b. Sensation does not return to areas that have undergone skin grafting.
c. Skin graft maybe exposed to moderate amounts of sunlight without changing pigmentation.
d. A split-thickness skin graft undergoes more shrinkage of its surface area immediately after harvesting than a full-thickness skin graft.
e. All of the above are true.

A

a. Secondary contraction is more likely to occur after adequate healing of a split-thickness skin graft than a full-thickness skin graft.

102
Q

Which one or more of the following recipient beds is unlikely to support a split-thickness skin graft?

a. Muscle with its overlying fascia intact.
b. Muscle without overlying fascia.
c. Tendon with its paratenon intact.
d. Tendon without its paratenon.
e. Nerve with its perineurium intact.

A

Answer: d. Tendon without its paratenon

In the other zones, the flexor tendons are surrounded by loose areolar (paratenon) tissue. Healing those parts devoid of a fibrous sheath is usually excellent because of the good blood supply from the paratenon.
- Townsend: Sabiston Textbook of Surgery, 18th ed.

103
Q

Patient had mastectomy followed by radiation. She came back with recurrence in the scar after one year. She with have wide excision. What is the best reconstruction procedure for her:

a. Tissue expander
b. Distal muscle flap
c. Local skin flap
d. Distal skin flap

A

Answer: b. Distal muscle flap
pedicled muscle flap (TRAM or LAT flap)
After radiation local skin not great–therefore expander or local skin flap won’t work.
Distal skin flap would be ok but might as well take muscle with it TRAM)

104
Q

A 65 year old male presents with painless hematuria. His history and physical examination are unremarkable. What is the most likely diagnosis?

a) bladder Ca
b) renal cell Ca
c) prostate Ca
d) kidney stone
e) trauma

A

a) bladder Ca

Causes of Asymptomatic Gross Hematuria by Incidence

  1. Acute Cystitis (23%)
  2. Bladder Cancer (17%)
  3. Benign Prostatic Hyperplasia (12%)
  4. Nephrolithiasis (10%)
  5. Benign essential Hematuria (10%)
  6. Prostatitis (9%)
  7. Renal cancer (6%)
  8. Pyelonephritis (4%)
  9. Prostate Cancer (3%)
  10. Urethral stricture (2%)
    - http://www.fpnotebook.com/Urology/Lab/GrsHmtr.htm

Most patients with hematuria do not have significant pathology, but this depends on the type of hematuria and underlying risk factors. One prospective study found that 61% of patients with hematuria had no finding after evaluation ( Khadra et al, 2000 ).
Gross hematuria is usually noticed by the patient and results in an office or emergency room visit. It is a common presenting symptom for patients with bladder or renal cancer ( Yun et al, 2004 ). The sensitivity of gross hematuria for bladder cancer is 83% and the positive predictive value is 22% ( Buntinx and Wauters, 1997 ). However, the studies generating these values were of low quality and based on referral populations. There are no studies available on the likelihood that family medicine patients with this finding will have serious pathology
- Rakel: Textbook of Family Medicine, 7th ed.

105
Q

A 75 year old man presents with pain in the left flank. He is assessed as having an obstructing stone in the ureter. Which of the following would require immediate attention:

a) temperature of 38.9
b) a stone measuring 10mm on CT scan
c) hydronephrosis
d) uncontrolled pain

A

a) temperature of 38.9

Infection associated with ureteral stones, or obstructed pyelonephritis, is a not uncommon and potentially life-threatening urologic emergency. Such patients are typically febrile and may present with signs of septic shock, such as hypotension. Urgent drainage of the obstructed portion of the urinary tract by either ureteral catheter or percutaneous nephrostomy is essential. A urine culture specimen from the obstructed segment will aid in directing antibiotic therapy. Definitive stone therapy should be delayed until urine cultures are negative and the patient has recovered completely.
- Wein: Campbell-Walsh Urology, 9th ed.

106
Q

A middle-aged ♂ patient could not urinate 8 hours postoperatively following an inguinal hernia repair. A Foley catheter was inserted and only drained 15cc. It was noted that it was difficult to inflate the balloon with more than 1cc of saline. What is the BEST next step?
A. Deflate the balloon and remove the catheter
B. Deflate the balloon and advance the catheter even further
C. Insert a suprapubic catheter
D. Increase the patient’s IVF rate

A

B. Deflate the balloon and advance the catheter even further

107
Q

A patient presents with post-obstructive diuresis. The mechanism can include all of the following EXCEPT:
A. High ANP
B. Increased extracellular volume
C. Efferent arteriole constriction and afferent dilation
D. Accumulation of urea

A

C. Efferent arteriole constriction and afferent dilation

The diuresis is typically a normal physiologic response to the volume expansion and solute accumulation occurring during obstruction. Sodium, urea, and free water are eliminated and the diuresis subsides after solute and fluid homeostasis is achieved.
The increased endogenous production and altered regulation of ANP induce a saline diuresis.
- Wein: Campbell-Walsh Urology, 9th ed.;

The mechanisms involved have been described in detail earlier and involve the combination of intrinsic damage to tubular salt, solute, and water reabsorption, as well as the effects of volume expansion, solute (e.g., urea) accumulation, and attendant increases in natriuretic substances such as ANP.
- Brenner: Brenner and Rector’s The Kidney, 8th ed

Possible mechanisms of post-obstructive diuresis include:

  • physiological excretion of the previously retained water, salt and urea
  • impaired sensitivity of collecting duct to ADH
  • inability to maintain a medullary solute concentration gradient due to increased medullary blood flow during obstruction
  • increased atrial natriuretic peptide (ANP) levels resulting in natriuresis and diuresis
  • Glynne, Allen, Pusey. Acute renal failure in practice. 2008
108
Q

A patient 3 days post nephrectomy developed fever and increased white count. The following day, fluid begins to drain from incision which contains an elevated amylase. The management should include:

a. Operate through previous incision
b. Percutaneously drain and put patient on tpn
c. Observe and place patient on tpn
d. Percutaneously drain while keeping the patient npo

A

Answer: Percutaneously drain and put patient on tpn

A particularly distressing postoperative complication is the development of a pancreatic fistula due to unrecognized intraoperative injury to the pancreas. This is usually manifested in the immediate postoperative period with signs and symptoms of acute pancreatitis and drainage of alkaline fluid from the incision. A CT scan of the abdomen demonstrates fluid collection in the retroperitoneum. Fluid draining from the incision should be analyzed for pH and the presence of amylase. Treatment involves percutaneous or surgical drainage of the fluid collection to avoid the development of a pancreatic pseudocyst or abscess ( Zinner et al, 1974 ; Spirnak et al, 1984). Most fistulas close spontaneously with the establishment of adequate drainage. Because the healing of a pancreatic fistula is usually a slow process, the patient is also supported with hyperalimentation. Surgical closure by excision of the fistulous track and construction of an anastomosis between the pancreas and a Roux-en-Y limb of the jejunum is only occasionally necessary in patients with prolonged drainage.
- Wein: Campbell-Walsh Urology, 9th ed.

109
Q

A 12 year old boy presents following 4 hours of acute onset right testicular pain. He denies any antecedent urinary symptoms. Formal examination of the right testicle is not possible because of significant tenderness. A urinalysis is negative. What is the MOST accurate statement regarding this situation?

a. There is a high likelihood that this condition affected the patient’s father and brothers
b. If symptoms have been present for greater than 12 hours, surgical intervention is not indicated
c. There is a high likelihood that the underlying abnormality which predisposes the patient to this condition is present in both testicles
d. No surgical intervention required until symptoms have been present for at least 12 hours
e. After 6-8 hours, the salvage rate markedly decreases, and it is near 0% at 12 hours

A

c.There is a high likelihood that the underlying abnormality which predisposes the patient to this condition is present in both testicles

You need to do do a bilateral orchidopexy

110
Q

A 12 year old boy awakes from night with sudden onset of right testicular pain. You are unable to adequately assess him clinically due to pain, but the right testicle appears raised and erythematous. You will:

a. Follow him clinically if he pain has been there for more than 12 hours
b. Examine under anesthesia
c. Find a similar history from brother and father
d. Serial examinations q hourly over next 12 hours
e. (Note: ultrasound with Doppler was not an option here)

A

b. Examine under anesthesia

Diagnosis of testicular torsion is made mainly by clinical suspicion. When uncertain, color Doppler ultrasound or a nuclear testicular scan may help with the diagnosis. In the case of epididymo-orchitis, Doppler ultrasound would demonstrate increased blood flow, and radionuclide can would show increased radionuclide activity. In the case of testicular torsion, however, Doppler would show no blood flow, and radionuclide scan would show poor radionuclide tracer uptake.
Immediate surgical exploration is indicated if testicular torsion is suspected. If treated within the first 4 to 6 hours of onset of symptoms, the chance of testicular salvage is high. During surgical exploration, the testicle is rotated to its normal position to restore blood flow to the testicle. If viable, orchiopexy of the affected and the contralateral testicle is completed. If the affected testicle is nonviable, orchiectomy of the affected testicle and orchiopexy of the contralateral side are performed.
- Townsend: Sabiston Textbook of Surgery, 18th ed.

111
Q

In a 30 years old male with acute testicular pain, the most common cause is:

a. Testicular torsion.
b. Epididymo-orchitis
c. Testicular hematoma
d. Testicular cancer

A

b. Epididymo-orchitis

Epididymitis is most common in adults, torsion is most common in children

112
Q

Which of the following is true regarding testicular torsion:

a. It can present as painless scrotal swelling
b. It is unheard of in the adult population (>30 yrs old)
c. It is most commonly associated with pain and a retracted testicle
d. Dysuria is a prominent feature
e. Doppler ultrasound demonstrates increase flow to the epidydimus

A

Answer: c. It is most commonly associated with pain and a retracted testicle

Although torsion of an undescended testis is rare, it should be considered in any child with abdominal or groin pain and an empty ipsilateral hemiscrotum.
- Wein: Campbell-Walsh Urology, 9th ed.

Presentation of testicular torsion is acute onset of testicular pain or swelling. Some patients may have episodic symptoms of pain suggestive of intermittent torsion. Physical examination may reveal a tender firm testicle, high riding testicle, horizontal lie of testicle, absent cremasteric reflex, and no pain relief with elevation of the testicle. The spermatic cord may appear thickened. The posteriorly positioned epididymis may be positioned differently.
In the case of epididymo-orchitis, Doppler ultrasound would demonstrate increased blood flow, and radionuclide can would show increased radionuclide activity. In the case of testicular torsion, however, Doppler would show no blood flow, and radionuclide scan would show poor radionuclide tracer uptake.
- Townsend: Sabiston Textbook of Surgery, 18th ed.

113
Q

A 47 year old female undergoes an elective vaginal hysterectomy. The woman’s hemoglobin drops from 125 pro-op to 80 post-op day 2. Other than feeling tired she has no complaints. An abdominal ultrasound shows a pelvic hematoma. The most appropriate management is:

a. observe
b. transfuse
c. ultrasound-guided drainage
d. surgical exploration and drainage
e. none of above

A

a. observe

When the hematoma is discovered early postoperatively, the patient should be returned to the operating room and, under sterile conditions, the wound opened, the hematoma evacuated, the responsible vessel found and ligated, and the wound closed primarily. If hematomas are discovered late, heat should be applied to the wound and the patient managed expectantly with the hope that the hematoma has not become contaminated. If the surgeon suspects the possibility of a postoperative hematoma because of predisposing factors or difficult hemostasis, a closed-suction drain should be used. The catheter should exit through a separate stab incision remote
from the primary wound.
- Swartz chp 11

114
Q

Which drug can you use in a patient with glaucoma?

a) Propofol ?
b) Oxybutanine
c) Propranolol

A

Propofol ? (this decreases IOP)
Oxybutanine (contraindicated)
Propranolol (actually indicated for glaucoma)

115
Q

The most sensitive finding of optic nerve injury after trauma is

a. visual field deficits
b. loss of papillary light reflex
c. loss of visual acuity
d. fundoscopic exam

A

Answer given: loss of visual acuity. Maybe loss of papillary light reflex?

Patients who have optic neuropathy have decreased visual acuity or visual field defects. A relative afferent pupillary defect – the sine qua non – of an optic neuropathy often is the ocular abnormality evident. If a relative afferent pupillary defect is not evident, the patient does not have a traumatic optic neuropathy, unless it is bilateral. Patients who have bilateral optic nerve dysfunction demonstrate light–near dissociation of their pupillary reactions. The near response is brisker than the pupillary response to light.
- Yanoff & Duker: Ophthalmology, 3rd ed.

Traumatic Optic Neuropathy: Workup
Traumatic optic neuropathy (TON) is based on a clinical diagnosis of optic nerve dysfunction supported by a recent history of trauma to the head. In an acute setting following trauma, the diagnosis may be delayed if the patient is unconscious and a formal visual acuity assessment cannot be performed. In a conscious, cooperative patient, the diagnosis of TON should be verified by testing the patient for an abnormal visual acuity, an ipsilateral afferent pupillary defect (APD), impairment of color vision, and a visual field defect on formal perimetry.
- http://emedicine.medscape.com/article/868129-diagnosis

116
Q
An achondroplastic dwarf is undergoing spinal fusion surgery and is placed in the prone position. In the recovery room, he is unable to see out of either eye. What is the MOST likely cause of his blindness?
A. Retinal detachment
B. Acute open angle glaucoma
C. Retinal ischemia
D. Occipital artery infarct
A

C. Retinal ischemia

The most common cause of perioperative retinal arterial occlusion is improper patient positioning resulting in external compression of the eye that produces sufficient intraocular pressure (IOP) to stop flow in the central retinal artery. It is the typical cause of the injury during spine surgery performed with the patient in the prone position
- Miller: Miller’s Anesthesia, 7th ed.

117
Q

65 year old female post strabismus surgery has subcutaneous emphysema all of the following are true except :

a. Surgical exploration
b. Observation
c. Chest tube insertion
d. None of the above
e. All of the above

A

Answer: surgical exploration

Huh?

The development of subcutaneous emphysema may cause marked distortion of the patient’s appearance, but there is no reason to “treat” the condition, except to take whatever steps are appropriate to stop the air leak. The source of the leak must be found, because some potential causes (esophageal perforation or major bronchial injury) require early intervention.

118
Q

Which of the following bacteria will flourish if

A

a. beta hemolytic strep

Skin grafts can be placed in the presence of 10^5 or less organisms (except beta-hemolytic strept).
- Morell Notes

Beta-hemolytic = Groups A-G

Despite its devastating effect on skin grafting, infection is the least common cause of graft failure. In addition to the obvious space-occupying effect of infection, a purulent collection actively separates the graft from its bed and destroys the newly formed vessels. The abundant proteolytic enzymes from inflammation and from the microorganisms are responsible for the lysis of the fibrin adhesion and the destruction of vascular connections. The most notable virulent organism that affects skin grafting is group A beta-hemolytic streptococcus (Streptococcus pyogenes), which rapidly destroys the skin graft and literally melts it away. Topical or systemic antibiotics effective against common microorganisms (i.e., staphylococcal, streptococcal, or Pseudomonas organisms) may be helpful in selective situations if administered early on, particularly for patients with diabetes or impaired immunosuppression or after a prolonged intraoperative time.
Immunocompetent individuals and do little to guarantee ultimate graft survival in questionable cases.
- Roberts: Clinical Procedures in Emergency Medicine, 3rd ed